Skip to main content

Full text of "كتب في علم الميكروبيولوجي"

See other formats


BLACKWELL'S 

KI.H;TH;r*l ll?l.ltHI?ir^l*'JM?l*Wrei 

MICROBIOLOGY 
VOL. I, 3E 



VIKAS BHUSHAN, MD 

University oi 'California, San Francisco, Class of 1991 
Series Editor, Diagnostic Radiologist 

VISHAL PALL, MBBS 

Government Medical College. Chandigarh. India. Class ol 1990 
Series Editor, U. of Texas, Galveston. Resident in Internal Medicine & 
Preventive Medicine 

TAO LE, MD 

University of California, San Francisco, Class of 1990 

HOANG NGUYEN, MD, MBA 

Northwestern University, Class oi 2001 



b 



Blackwell 
Science 






CONTRIBUTORS 

Sandra Mini 

University of Texas Medical Branch. Class of 201)2 

Beth Ann Fair, MD 

Eastern Virginia Medical School, Resident in F.mergencv Medicine 

Kristen Lem Mygdal, MD 

I Diversity of Kansas School of Medicine. Resident in Radiology 

Mae Sheikh-Ali, MD 

University o( Damascus, Syria, ( II ass of 1099 

Shaiin Patel, MD 

M<< .raw Medical Center, Northwestern University, Resident in Internal 
Medicine 

Jose M. Fierro, MD 

la Salle University, Mexico City 

FACULTY REVIEWER 

Warren Levinson, MD. PHD 

Professor of Microbiology and Immunology. UCSF School of Medicine 



© 2002 by Blackwell Science. Inc. 

Editorial Offices: 

Commerce Place. 350 Main Street, Maiden, 
Massachusetts 02148. USA 

Osney Mead. Oxford OX2 OH.. Kngland 
25 John Street, London WC1N 2BS. England 
23 Ainslie Place, Edinburgh EH3 6AJ, Scotland 

54 University Street, Carlton, Victoria 3053, 
Australia 

Other Editorial Offices: 

Blackwell Wissenschalls-Y'erlag GmbH. 

Imm hirstendamm 57, 10707 Berlin. Germany 
Blackwell Science KK. MG Kodcnmacho Building. 

7-10 Kodcnmacho Nihombashi, Ghuo-ku, 

Tokyo 104. Japan 
Iowa State University Press. A Blackwell Science 

Company, 2121 S. Stale Avenue. Ames. Iowa 

50014-8300, USA 

Distributors: 

The Americas 

Blackwell Publishing 

c/o AIDC 

P.O. Box 20 

50 Winter Spoil Lane 

Williston, VT 0549541020 

(Telepht>ne orders: 800-216-2522; 

fax orders: 802-864-7626) 
Australia 

Blackwell Science Ptv. I. id. 
54 University Street 
Carlton. Victoria 3053 
(Telephone orders: 03-9347-0300: 

fax orders: 03-9349-3016) 
Outside The Americas and Australia 

Blackwell Science. Ltd. 

c/o Mansion Book Services. I .td. 

P.O. Box 269 

Abingdon 

OxonOX14 4YN 

England 

(Telephone orders: 444)1235-465500; 

fax orders: 44-0 1 235-465555 ) 



Acquisitions: Laura DeYoung 

Development: Amy Nuttbrock 

Production: l.orna Hind and Shawn Girsberger 

Manufacturing: Lisa Flanagan 

Marketing Manager: Kathleen Mulcahy 

( lover design bv Leslie Haiines 

Interior design by Shawn Girsberger 

Typeset by TechBooks 

Printed and bound by Capital City Press 

Blackwell's Underground Clinical Vignettes: 
Microbiology I, 3e 

ISBN 0-632-04547-7 

Printed in the United Stales oi America 
02 03 04 05 5 4 3 2 1 

The Blackwell Science logo is a trade mark of 
Blackwell Science Lid., registered at the Linked 
Kingdom Trade Marks Registry 

Library of Congress Calaloging-in-Publication Data 

Bluishan. Vikas. 

lilackwell's underground clinical vignettes. 

Microbiology Author, Vikas Bliiislian. - 3rd ed. 

p. ; cm. - (Underground clinical vignettes) 
Rev. ed. of: Microbiology / Vikas Bhushan ... let at.]. 
2nd ed. cl999-. ISBN 0-632-04547-7 (alk. paper) 

1. Medical microbiology - Case studies. 

2. Physicians - Licenses - United Slates - 
Examinations- Study guides. 

[DNLM: 1. Microbiology - Case Report. 
2. Microbiology - Problems and Exercises. 
QW 18.2 B575b 2002) I. Title: Microbiology. 

II. ride: Underground clinical vignettes. 
Microbiology. III. Microbiology. IV. Title. V. Series. 

QR46 .B465 2002 

0UV.oro70-dc21 

200100-1032 



All rights reserved. No part of this book may be 
reproduced in any form or by any electronic or 
mechanical means, including information storage 
and retrieval systems, without permission in writing 
trom the publisher, except by a reviewer who may 
quote brief passages in a review. 






Notice 

I lie .111 1 hr >is nl ill is Mil 111 in- have taken care thai [he information contained herein is an urate and compali- 
hle with the standards generally accepted at the lime Of publii ation. Nevertheless, it is difficult to ensure 
that all the information given is entirely accurate For all circumstances. The publisher and authors do not 
guarantee die contents ol this book and disclaim an) liability, loss, or damage incurred as a consequent e, 
directly or indirectly, of the use and application ol any of the contents ol this volume. 



CONTENTS 



Acknowledgments x 

Preface to the 3rd Edition xiii 

How to Use This Book xvi 

Abbreviations xvii 



Cardiology 



Dermatology 



ENT/ Ophthalmology 



Gastroenterology 



Acute Bacterial Endocarditis 

Myocarditis — Viral 

Pericarditis — Acute 

Prosthetic Valve Endocarditis 

Subacute Bacterial Endocarditis 

Cellulitis 

Erysipelas 

Erysipeloid 

Erythema Infectiosum 

Impetigo 

Molluscum Contagiosum 

Pityriasis Versicolor 

Roseola Infantum 

Scalded Skin Syndrome 

Tinea Corporis (Ringworm) 

Urticaria 

Acute Conjunctivitis 

Acute Sinusitis 

Allergic Rhinitis (Hay Fever) 

Common Cold (Viral Respiratory Infection) 

Herpes Zoster Ophthalmicus 

HSV Keratitis 

Otitis Externa 

Otitis Media 

Pharyngitis — Adenovirus 

Pharyngitis — Streptococcal 

Fitz-Hugh-Curtis Syndrome 

Gastroenteritis — Staphylococcus aureus 

Hepatitis A 

Hepatitis B— Acute 

Hepatitis C — Chronic Active 

Hookworm 

Necrotizing Enterocolitis 

Neutropenic Enterocolitis 

Peptic Ulcer Disease (H. pylori) 

Pinworm Infection 

Rotavirus Diarrhea in Infants 



m 



m 
m 



Kil 



Genetics 
Hematology/ 'Oncology 



Infectious Disease 



Salmonella Food Poisoning 


IB 


Spontaneous Bacterial Peritonitis 


m 


Traveler's Diarrhea 


B81 


Vibrio parahaemolyticus Food Poisoning 


Efl 


Vibrio vulnificus Food Poisoning 


m 


Whipple's Disease 


EO 


Yersinia Enterocolitis 


ca 


Chediak-Higashi Syndrome 


Eg 


Anemia — Aplastic Crisis (Parvovirus B19) 


E9 


Anemia — Diphyllobothrium latum 


E9 


Graft-Versus-Host Disease 


PEl 


Hemolytic-Uremic Syndrome (HUS) 


m 


Actinomycosis 


B3 


Acute Bronchiolitis 


Bl 


Acute Rheumatic Fever 


m 


African Trypanosomiasis 


m 


AIDS-Related Complex (ARC) 


Ea 


Amebic Colitis 


m 


Amebic Liver Abscess 


m 


Amebic Meningoencephalitis 


Bl 


Anthrax 


m 


Aspergillosis 


m 


Aspergillosis— Allergic Bronchopulmonary 


BQ 


Aspiration Pneumonia with Lung Abscess 


Bl 


Atypical Mycobacterial Infection 


m 


Bacillus cereus Food Poisoning 


m 


Bartonellosis 


m 


Blastomycosis 


m 


Botulism 


m 


Brucellosis 


m 


Campylobacter Enteritis 


E3 


Candidiasis 


BCl 


Cat-Scratch Disease 


Bl 


Chagas' Disease 


Bl 


Chlamydia Pneumonia 


m 


Chlamydia trachomatis 


m 


Cholera 


w 


CMV— Congenital 


m 


CMV Pneumonitis 


m 


CMV Retinitis 


m 


Coccidioidomycosis 


m 


Colorado Tick Fever 


m 


Croup 


ETil 


Cryptosporidiosis 


Bl 



Diphtheria 

Echinococcosis 

Ehrlichiosis 

Endemic Typhus 

Epidemic Typhus 

Epiglottitis 

Gas Gangrene — Traumatic 

Giardiasis 

Gonococcal Ophthalmia Neonatorum 

Gonorrhea 

Granuloma Inguinale 

H. influenzae in a COPD Patient 

Hantavirus Pulmonary Syndrome EQ 

Hemorrhagic Fever— Crimean-Congo 158 

Hemorrhagic Fever — Dengue 

Hemorrhagic Fever — Ebola Virus fcfri 

Hemorrhagic Fever — Renal Syndrome EE1 

Herpangina 

Herpes Genitalis 

Herpes Zoster (Shingles) 






ACKNOWLEDGMENTS 



Throughout the production of this book, we have had the sup- 
port of man\' friends and colleagues. Special thanks to our sup- 
port team including Ann Gupta, Andrea Fellows, Anastasia 
Anderson, Srishti Gupta, Mona Pall, Jonathan Kitsch and Ghirag 
Amin. For prior contributions we thank Gianni L.e Nguyen, 
Tarun Mathur, Alex ( .1 imm. Souia Santos and Elizabeth 
Sanders. 

We have enjoyed working with a world-class international pub- 
lishing group at Blackwcll Science, including I. aura DeYoung, 
Amy Nuttbrock, Lisa Flanagan, Shawn Girsberger, I.orna Hind 
and Gordon Tibbitts. For help with securing images for the 
entire series we also thank Lee Mai tin. Kristopher Jones, Tina 
Pani/./.i and Peter Anderson at the University of Alabama, the 
Aimed Forces Institute of Pathology, and many ol our fellow 
Blackwell Science authors. 

For submitting comments, corrections, editing, proofreading, 
and assistance across all of the vignette titles in all editions, we 
( ollec liveh thank: 

Tara Adamovich, Carolyn Alexander, Kris Alden, Henry E. Aryan, 
l.vmnan Bacolor, Natalie Barteneva. Dean Bartholomew, 
Debashish Behera, Sumit Bhatia. San jay Bindra. Dave Brinton, 
|ulianne Brown. Alexander Brownie. Tamara Callahan, David 
Canes, Bryan Casey, Aaron Caughey, Hebert Chen, Jonathan 
Cheng, Arnold Cheung, Arnold Chin. Simion Chiosea, Yoon Cho, 
Samuel Chung, Grelchen Conant. Vladimir Coric, Christopher 
Cosgrove, Ronald Cowan, Karckin R. Cunningham, A. Sean 
Dalley. Rama Dandamudi. Sunil Das, Ryan Armando Dave, John 
David, Emmanuel de la Cruz, Robert DeMello, Navneet Dhillon. 
Sharmila Dissanaike. David Donson, Adolf Etchegaray, Alea 
Eusebio, Priscilla A. Frase. David Frenz. Kristin Gaumer, 
Yohannes Gebreeg/.iabher, Anil Gehi, Tony George, L.M. 
Gotanco, I'arul Goval. Alex Grimm. Rajeev Gupta, Ahmad 
I lalim. Sue I [all, David Hasselbacher, Tamra Heimert, Michelle 
Higley, Dan Hoit, Eric Jackson, Tim Jackson, Sundarjayaraman, 
Pei-Ni (one. Aarchan Joshi. Rajni K. Jutla, Faiyaz Kapadi, Seth 
Karp. Aaron S. Kesselheim, Sana Khan. Andrew Pin-wei Ko. 
Francis Kong, Paul Konitzky. Wanen S. Krackov, Benjamin H.S. 
I.au. .Ann LaCasce, Connie Lee. Scon Lee. Guillcrmo l.ehmann. 
Kevin Leung'. Paul Levett. Warren Levinson. Eric Lev. Ken Lin. 



Pavel Loban ov, J. Mark Maddox, Aram Mardian. Sarair Mehta, 
Gil Melmed,Joe Messina, Robert Mosca, Michael Murphy, Vivek 
Nandkami. Siva Naraynan, Carvell Nguyen, l.inh Nguyen, 
Deanna Nobleza, Craig Nodurft, George Noumi, Darin T. 
Okuda, Adam L. Palance, Paul Paniphrus, Jinha Park, Sonny 
Patel. Ricardo Pietrobon, Riva L. Rahl, Aashita Randeria. 
Rachan Reddy, Beatriu Reig, Marilou Reyes, Jeremy Richmon, 
Tai Roe. Rick Roller, Rajiv Roy, Diego Ruiz, Anthony Russell. 
Sanjay Sahgal, Urmimala Sarkar, John Schilling. Isabel] Schmitt, 
Daren Schuhmacher, Sonal Shah, Fadi Abu Shahin, Mae Sheikh- 
Ali, Edie Shen, JusUn Smith, John Slulak. Lillian Su, Julie 
Sundaram, Rita Suri, Seth Sweetser. Antonio Talayero, Merita 
Tan, Mark Tanaka. Eric Taylor, Jess Thompson, Inch Trehan, 
Raymond Turner, Okafo Uchcnna, Eric Uyguanco. Richa 
V'arma, John Wages, Alan Wang, Eunice Wang, Andy Weiss, Amy 
Williams. Brian Yang, I lain Zaky, Ashraf Zaman and David Zipl. 

For generously contributing images 10 the entire Underground 
Clinical Vignette Step 1 series, we collectively thank the stair at 
Blackwell Science in Oxford, Boston, and Berlin as well as: 

• Axford, J. Medici iii: Osney Mead: Blackwell Science Ltd, 1996. 
Figures 2.14, 2.15, 2.16, 2.27. 2.28. 2.31, 2.35. 2.36. 2.38, 2.43, 
2.65a, 2.65b, 2.65c, 2.103b, 2.105b. 3.20b, 3.21, 8.27, 8.27b, 
8.77b, 8.77c, 10.81b, 10.96a, 12.28a, 14.6, 14.16, 14.50. 

• Bannister B, Begg N, Gillespie S. Infectious Disease, 2 wl Edition. 
Osney Mead: Blackwell Science Ltd, 2000. Figures 2.8, 3.4, 
5.28, 18.10, W5.32, W5.6. 

• Berg D. Advanced Clinical Skills and Physical Diagnosis. 
Blackwell Science Ltd., 1999. Figures 7.10, 7.12, 7.13, 7.2, 7.3, 
7.7.7.8,7.9,8.1.8.2,8.4,8.5,9.2, 10.2, 11.3,11.5, 12.6. 

• Cuschieri A, Hennessy TPJ, Greenhalgh RM, Rowley DA, 
Grace PA. Clinical Surgery. Osney Mead: Blackwell Science 
Ltd, 1996. Figures 13.19, 18.22, 18.33. 

• Gillespie SH, Bamford K. Medical Microbiology and Infection al n 
Giant*. Osney Mead: Blackwell Science Ltd, 2000. Figures 20, 23. 

• Ginsberg L. Lecture Notes on Neurology. 7' Edition, Osney Mead: 
Blackwell Science Ltd, 1999. Figures 12.3, 18.3, 18.3b. 

• Elliott T. Hastings M. Desselberger V. Lecture . Wiles on Medical 
Microbiology, >"' Edition. Osney Mead: Blackwell Science Ltd, 
1997. Figures 2, 5, 7, 8, 9, 11, 12, 14. 15, 16. 17, 19, 20, 25, 26, 
27, 29, 30, 34, 35, 52. 






• Mehta AB, Hoffbrand AV. Haonalolooy at a Glance. Osney 
Mead: Blackwell Science Ltd, 2000. Figures 22.1. 22.2, 22.3. 

Please lei us know if your name has been missed or misspelled 
and we will be happy to make the update in the next edition. 



PREFACE TO THE 3RD EDITION 



We were very pleased with the overwhelmingly positive student 
feedback for the 2nd edition of our Underground Clinical Vignettes 
scries. Well over 100,000 copies of the UCV books are in print 
and have been used by students all over the world. 

Over the last two years we have accumulated and incorporated 
over a thousand "updates" and improvements suggested by you, 
our readers, including: 

• many additions of specific boards and wards testable content 

• deletions of redundant and overlapping cases 

• reordering and reorganization of all cases in both series 

• a new master index by case name in each Atlas 

• correction of a few factual errors 

• diagnosis and treatment updates 

• addition of 5-20 new cases in every book 

• and the addition of clinical exam photographs within UCV- — 

Anatomy 

Arid most important of all. the third edition sets now include 
two brand new COLOR ATLAS supplements, one for each 
Clinical Vignette series. 

• The UCV-Basic Science Color Alias (Step 1) includes over 250 
color plates, divided into gross pathology, microscopic pathol- 
ogy (histology)- hematology, and microbiology (smears). 

• The UCV-Clinical Science Color Atlas {Step 2) has over 125 color 
plates, including patient images, dermatology, and fundus- 
copy. 

Each atlas image is descriptively captioned and linked to its cor- 
responding Step 1 case, Step 2 case, and/or Step 2 MiniCase. 









How Atlas Links Work: 



Step 1 Book Codes are: A = Analom) 



BS = Bili.uioi.il Science 
BC = Bini licinisii \ 
Ml = Microbiology, Vol. 1 
Ml' - Microbiology, Vol. II 
PI = Pathophysiology, Vol. I 
P2 = Pathophysiology, Vol. II 
P I Pathophysiology, Vol, 111 
PH = Pharmacology 



Step 2 Book Codes are: ER = Emergency Medicine 

IM I = Internal Medicine. Vol. I 
IM2 = Internal Medicine, Vol. U 

NHl ' - Neurology 
OB = OB/GYN 
PED Pediatrics 
SUR = Surgei \ 
PSY= Psvchialrx 
MC = MiniCasc- 




Case Number 




M-P3-032A 



Indicates Type of Image: II = Hematology 
M = Microbiolog) 
I'c. - ( »ross Pathology 
I'M - Microscopii Patholog) 




ER-035A, ER-035B 



Indicates UCV1 or UCV2 Series 






• If the Case number (032. 035. etc.) is not followed by a letter, 
then there is only one image. Otherwise A, B. C, D indicate 
up to 4 images. 

Bold Faced Links: In order to give you access to the largest 
□umber of images possible, we have chosen to cross link the 

Step 1 and 2 series. 

• If ihe link is bold-faced I h is indicates that the link is direct 
(i.e.. Step 1 Case with the Basic Science Step 1 Atlas link). 

• If the link is not bold-faced this indicates that the link is indi- 
rect (Step 1 case with Clinical Science Step 2 Atlas link or vice 
versa). 

We have also implemented a few structural changes upon your 
request: 

• Each current and future edition of our popular First Aid far 
the USMLESle/i 1 (Appleton & Lange/McGraw-Hill) and First 
Aid for the USMLESlej/ 2 (Appleton & Lange/McGraw-Hill) 
book will be linked to the corresponding UCV case. 

• We eliminated UCV — ► First Aid links as they frequently 
become out of date, as the First Aid books are revised yearly. 









• The Color Atlas is also specially designed l< ir quizzing — 
captions are descriptive and do not give away the case name 
directly. 

We hope the updated UCV series will remain a unique and well- 
integrated study tool that provides compact clinical correlations 
to basic science information. They are designed to be easy and 
fun (comparatively) to read, and helpful for both licensing 
exams and the wards. 

We invite your corrections and suggestions for the fourth edi- 
lion of these books. For the first submission of each factual cor- 
rection or new vignette that is selected for inclusion in the 
fourth edition, you will receive a personal acknowledgement in 
die revised book. If you submit oxer 21) high-quality corrections, 
additions or new vignettes we will also consider inviting you to 
become a "Contributor" on the book of your choice. Lf you are 
interested in bet oming .1 potential "Contribitti >r" or "Author" 
on a future UCV book, or working with our team in developing 
additional books, please also e-mail us your CV/resume. 

We prefer that you submit corrections or suggestions via 
electronic mail to UCVteam@yahoo.com. Please include 
"Underground Vignettes" as the subject of vour message. If you 
do not have an ess to e-mail, use the following mailing address: 
Blackwell Publishing, Attn: UCV Editors, 350 Main Street, 
Maiden, MA 02148, USA. 

1/7,7/ s Hh iish tin 
\ "tshal Pall 
Too Lf 
October 2001 









HOW TO USE THIS BOOK 



This series was originally developed to address the increasing 
number of clinical vignette questions on medical examinations. 
including the USMLE Step 1 and Step 2. It is also designed to 
supplement and complement the popular First Aid for the 
I 'SMLE Step 1 (Appleton & Lange/McGraw Hill) and First Aid 
for the l 'SMLE Step 2 (Appleton & Lange/McGraw Hill). 

Each UCV 1 book uses a series of approximately 100 "supra- 
prototypical" cases as a way to condense testable facts and 
associations. The clinical vignettes in this series are designed to 
incorporate as many testable facts as possible into a cohesive 
and memorable clinical picture. The vignettes represent 
composites drawn from general and specialty textbooks, 
reference books, thousands of USMLE style questions and the 
personal experience of die authors and reviewers. 

Although each case lends to present all the signs, symptoms. 
.iikI dia^nosti< lindings foi a parti< ulal illness, patients gener- 
ally will not present with such a "complete" pichire either clini- 
cally or on a medical examination. Cases are not meant to 
simulate a potential real patient or an exam vignette. All the 
boldfaced "buzzwords" are for learning purposes and are not 
necessarily expected to be found in any one patient with the 
disease. 

Definitions of selected important terms are placed widiin the 
vignettes in (SM \t I < IAPS) in parentheses. Other parenthetical 
remarks often refer to the pathophysiology or mechanism of 
disease. The format should also help students learn to present 
cases succinctly during oral "bullet" presentations on clinical 
rotations. The cases are meant to serve as a condensed review, 
not as a primary reference. The information provided in this 
book has been prepared with a great deal of thought and care- 
ful research. This book should not, however, be considered as 
your sole source of information. Corrections, suggestions and 
submissions of new cases are encouraged and will be acknowl- 
edged and incorporated when appropriate in future editions. 



ABBREVIATIONS 



5-ASA f)-aniinosalicylic acid 

ABGs arterial blood gases 

ABVD adriamycin/bleomycin/vincristine/dacarbazine 

ACE angiotensin-con verting enzyme 

ACTH adrenocorticotropic hormone 

ADH antidiuretic hormone 

AFP alpha fetal protein 

AI aortic insufficiency 

AIDS acquired immunodeficiency syndrome 

ALL acute lymphocytic leukemia 

ALT alanine transaminase 

AML acute myelogenous leukemia 

AN \ antinuclear antibody 

ARDS adult respiratory distress syndrome 

ASD atrial septal defect 

ASO anti-streptolysin O 

AST aspartate transaminase 

AV arteriovenous 

BE barium enema 

BP blood pressure 

111 \ blood urea nitrogen 

CAD coronary artery disease 

CALLA common acute lymphoblastic leukemia antigen 

CBC complete blood count 

CHF congestive heart failure 

CK creatine kinase 

CLL chronic lymphocytic leukemia 

CML chronic myelogenous leukemia 

CMV cytomegalovirus 

CNS central nervous system 

COPD chronic obstructive pulmonary disease 

CPK creatine phosphokinase 

CSF cerebrospinal fluid 

CT computed tomography 

( 'V A cerebrovascular accident 

CXR chest x-ray 

DIC disseminated intravascular coagulation 

DIP distal interphalangeal 

DKA diabetic ketoacidosis 

DM diabetes mellitus 

I) I Rs deep tendon reflexes 

DVT deep venous thrombosis 






I .UN Epstein— Barr virus 

ECG electrocardiography 

Echo echocardiography 

EF ejection fraction 

1- < .1 ) esophagogastroduodenoscopy 

EMG electromyography 

ERCP endoscopic retrograde cholangiopancreatography 

KSR er\ throe) le sedimentation rate 

FEV forced expiratory volume 

FNA line needle aspiration 

FTA-ABS fluorescent treponemal antibody absoi ption 

FVC forced vital capacity 

GFR glomerular filtration rate 

GH growth hormone 

(il gastrointestinal 

GM-CSF granulocyte macrophage colony stimulating 

factor 

GU genitourinary 

HA\" hepatitis A virus 

hcG human chorionic gonadotrophin 

HEENT head, eves, ears, nose, and throat 

HIV human immunodeficiency virus 

HLA human leukocyte antigen 

IIP1 history of present illness 

HR heart rate 

HRIG human rabies immune globulin 

1 IS hereditary spherocytosis 

ID/CC identification and chief complaint 

IDDM insulin-dependent diabetes mellitus 

Ig immunoglobulin 

IGF insulin-like growth factor 

IM intramuscular 

JVP jugular venous pressure 

KUB kidneys/ ureter/bladder 

I.DH lactate dehydrogenase 

LES lower esophageal sphincter 

LFTs liver function tests 

LP lumbar puncture 

LV left ventricular 

I VI I left ventricular hypertrophy 

I aics electrolytes 

MCHC mean corpuscular hemoglobin concentration 

MCV mean corpuscular volume 

MEN multiple endocrine neoplasia 



MIS US monoclonal gammopathy of undetermined 

significance 

\l 1 1< major histocompatibility complex 

MI myocardial infarction 

MOPP mcchlorcthamine/vincristine (Oncovorin)/ 

procai ba/ine/ prednisone 

MR magnetic resonance (imaging) 

\ 1 i I . non-Hodgkin's lymphoma 

NIDDM non-insulin-dependent diabetes mellitus 

NPO nil per os (nothing by mouth) 

NSAID nonsteroidal anti-inllammalory drug 

PA posteroanterior 

PIP proximal interphalangeal 

I'li.s peripheral blood smear 

PE physical exam 

PFTs pulmonary function tests 

I'M I point of maximal intensity 

PMN polymorphonuclear leukocyte 

PT prothrombin lime 

I'l ( A percutaneous transluminal angioplasty 

PTII parathyroid hormone 

Pi T partial thromboplastin time 

PUD peptic ulcer tlisease 

RB< : red blood cell 

RPR rapid plasma reagin 

RR respiratory rate 

RS Reed-Sternberg (cell) 

RV tight ventricular 

RVH r 'g nl ventricular hypertrophy 

SBFT small bowel follow-through 

SIADH syndrome of inappropriate secretion of ADH 

SLE systemic lupus erythematosus 

S I I ) sexually transmitted disease 

TFTs thyroid function tests 

I PA tissue plasminogen activator 

TSH thyroid-stimulating hormone 

TIBC total iron-binding capacitv 

TIPS transjugular intrahepatic portosystemic shunt 

TPO thyroid peroxidase 

TSH thyroid-stimulating hormone 

TTP thrombotic thrombocytopenic purpura 

UA urinalysis 

UCSI upper GI 

I S ultrasound 






\ I ) \i I Venereal Disease Research Laboratory 

VS vital signs 

VT ventricular Uichvcardia 

\\1'.( white blood cell 

W'I'W Wolff-Parkinson-White (syndrome) 

XR x-ray 






ID/CC A 25-year-old IV drug abuser presents with a high fever with 

chilis, malaise, a productive cough, hemoptysis, and right-sided 
pleuritic chest pain. 

HPI He also reports multiple skin infections at injection sites. 



PE VS: fever. PK: stigmata of intravenous drug abuse at multiple 
injection sites; skin infections: thrombosed peripheral veins; 
splenomegaly and pulsatile hepatomegaly; ejection systolic 
murmur, increasing with inspiration, heard in tricuspid area. 

Labs CBC; normochromic, normocytic anemia. UA: microscopic 
hematuria. Blood culture yields Staphylococcus aureus. 

Imaging Echo: presence of vegetations on tricuspid valve and tricuspid 
incompetence. CXR: consolidation. 

Treatment High-dose intravenous penicillinase-resistant penicillin in 

combinalion with an aminoglycoside. If the isolated .V. aureus 
strain is methicillin resistant, vancomycin is the drug of choice. 

Discussion In drug addicts, the tricuspid valve is die site of infection more 
frequently (55%) than the aortic valve (35%) or the mitral valve 
(30% ): these findings contrast markedly with the rarity of right- 
sided involvement in cases of infective endocarditis (hat are not 
associated with drug abuse. Staphylococcus aureus is responsible 
for the majority of cases. Certain organisms have a predilection 
for particular valves in cases of addict-associated endocarditis; 
for example, enterococci, other streptococcal species, and 
non-albicans Candida organisms predominantly affect the valves 
of die left side of the heart, while S. aureus infects valves on both 
the right and the left side of the heart. Pseudomonas organisms 
are associated with biventricular and multiple-valve infection in 
addicts. Complications of endocarditis include congestive heart 
failure, ruptured valve cusp, myocardial infarction, and 
myocardial abscess. 



> 
o 



O 

CD 

-< 



Atlas Link 



PG-M1-001 



ACUTE BACTERIAL ENDOCARDITIS 






ID/CC A 25-year-old male complains of increasing shortness of breath 
and ankle edema thai have progressively worsened over the past 

'_' weeks. 

HPI He also complains of fatigue, palpitations, and low-grade fever. 
His symptoms followed a severe URL He denies any history <>l 
joint pain or skin rash (vs. rheumatic fever). 

PE JVP elevated; pitting pedal edema; fine inspiratory crepitations 
heard at both lung bases; mild hepatosplenomegaly. 

Labs ASO titers not elevated. CBC: lymphocytosis. EGG: first-degree 
AV block. ESR elevated; increased tilers of antibodies to 
coxsackievirus demonstrated in serum. 

Imaging CXR: cardiomegaly and pulmonary edema. Echo: dilated 
cardiomyopathy with low ejection fraction. 

Gross Pathology Dilated heart with foci of epicardial, myocardial, and 
endocardial petechial hemorrhages. 

Micro Pathology Endomyocardial biopsy reveals diffuse infiltration by 

mononuclear cells, predominantly lymphocytes; focal fibrosis. 

Treatment Manage congestive heart failure and arrhythmias: cardiac 
transplant in intractable cases. 

Discussion Coxsackie B is most often implicated in viral myocarditis. 

Nonviral causes of myocarditis include bacteria such as Borrelia 
burgdorferi (Lyme disease I, parasites such as Trypanosoma rruzi 
(Chagas' disease), hypersensitivity reaction (systemic lupus 
erythematosus, drug reaction), radiation, and sarcoidosis; may 
also be idiopathic (giant cell myocarditis). 

Atlas Link LZLLLJ M-Ml-002 



MYOCARDITIS— VIRAL 



ID/CC A 35-year-old male complains of fever, nonproductive cough, 
and chest pain. 

HPI Ht- slates llial (he chest pain developed after lie had a severe 
cold for 1 week. He describes the pain as severe, crushing, and 
constant over the anierior rhesi and adds that it worsens with 
inspiration and is relieved by sitting up and bending forward. 

PE VS: low-grade fever; sinus tachycardia. PE: triphasic pericardial 

friction rub (systolic and diastolic components followed by a third 
component in late diastole associated with atrial contraction): 
elevated JVP; inappropriate increase in JVP with inspiration 

(RussMAt'i.'s Mt.\): pulsus paradoxus may also be seen. 



> 

o 



o 

-< 



Labs Moderately elevated transaminases and I.f)H: elevated ESR; 

serum CPK-MB normal. CBC: neutrophilic leukocytosis. ECG: 
diffuse ST-segment elevation (vs. myocardial infarction); 
PR-segment depression. 

Imaging Echo: pericardial effusion. CXR: apparent cardiomegaly (due to 
effusion). 



Gross Pathology 



In long-standing cases, pericardium may become librotic. 
scarred, and calcified. 



Micro Pathology 



Pericardial biopsy reveals signs of acute inflammation with 
increased leukocytes, vascularity, and deposition of fibrin. 



Treatment Analgesics for pain; steroids in resistant cases; indomeihacin: 
surgical stripping of scarring in severe cases. 



Discussion Acute pericarditis is commonly idiopathic. Known infectious 

causes include coxsackievirus A and B, tuberculosis, staphylococcal 
or pneumococcal infection, amebiasis, or actinomycosis; 
noninfectious causes include chronic renal failure. 
collagen-vascular disease (systemic lupus erythematosus, 
scleroderma, and rheumatoid arthritis), neoplasms, myocardial 
infarction, and trauma. Long-term sequelae include chronic 
constrictive pericarditis. 

Atlas Link mem i PG-Mi-003 



PERICARDITIS— ACUTE 






ID/CC A 64-year-old male presents with rapidly progressive dyspnea 
and fever. 

HPI He has a history of orthopnea and paroxysmal nocturnal 

dyspnea and also reports pink, frothy sputum (HEMOPTYSIS). 
One month ago he underwent a bioprosthetic valve replace- 
ment for calcine aortic stenosis. He is not hypertensive and has 
never had oven cardiac failure in the past. 

PE VS: fever; hypotension. PE: bilateral basal inspiratory crackles 
heard: cardiac auscultation suggestive of aortic incompetence 
(early diastolic murmur heard radiating down left sternal edge). 

Labs CBC: normochromic, normocyiic anemia. Three consecutive 
blood cultures yield coagulase-negative Staphylococcus 
epidermidis: si rain found to be methicillin resistant. 

Imaging CXR (PA view): suggestive of pulmonary edema. Echo: confirms 
presence ol prosthetic aoriic valve dehiscence leading to 

incompetence and poor left ventricular function. 

Treatment High-dose parenteral antibiotics — vancomycin (drug of choice 
For meihicillin-resistant .S". aureus), gentamicin, and oral 
rifampiciu; surgical replacement of damaged prosthetic valve; 
prophylactic antibiotics (amoxicillin) for patients receiving 
oral/ dental treatments to prevent transient bacteremia. 

Discussion Prosthetic valve endocarditis is subdivided into two categories: 
early prosthetic valve endocarditis (EPVE), which becomes 
clinically manifest within 60 days after valve replacement (most 
commonly caused by Staphylococcus epidermidis, followed by 
gram-negative bacilli and Candida), and late prosthetic valve 
endocarditis (LPVE). which is manifested clinically more than 
60 days after valve replacement (most commonly caused by 
viridans streptococci). 



PROSTHETIC VALVE ENDOCARDITIS 



ID/CC A 25-year-old female complains of low-grade fever and myalgia 
of 3 weeks' duration. 

HPI She has a history of rheumatic heart disease (RHD). One month 
ago, she underwent a dental extraction and did not take the 
antibiotics that were prescribed for her. 

PE VS: fever. PE: pallor; small peripheral hemorrhages with slight 
nodular character (Janeway 1-F.sions); small, tender nodules on 
finger and toe pads (Osler's nodes); subungual linear streaks 
(splinter hemorrhages); petechial hemorrhages on conjunctiva, 
oral mucosa, and upper extremities; mild splenomegaly; apical 
diastolic murmur on cardiovascular exam; fundus exam shows 
oval retinal hemorrhages (Roth's spots). 

Labs CBC/PliS: normocytic, normochromic anemia. UA: microscopic 
hematuria. Growth of penicillin-sensitive Streptococcus viridans 
on five of six blood cultures. 

Imaging Echo: vegetal ions along atrial surface of mitral valve. 



Gross Pathology 



Micro Pathology 



Treatment 



Discussion 



Embolism from vegetative growths on valves may embolize 
peripherally (left-sided) or to the lung (right-sided). 

Bacteria form nidus of infection in previously scarred or 
damaged valves; bacteria divide unimpeded once infection takes 
hold with further deposition of fibrin and platelets; peripheral 
symptoms such as Osier's nodes are believed to result from 
deposition of immune complexes. 

IV 3-lactamase-resistant penicillin and gentamicin; bacteriostatic 
treatments ineffective. 

S. viridans is the most common cause of subacute infective 
endocarditis, while Staphylococcus aureus is the most common 
cause of acute bacterial endocarditis. Prophylactic antibiotics 
should be given to all RHD patients before any dental procedure. 
The disease continues to be associated with a high mortality rate. 



Atlas Link [~1 TT PG-M1-005 



C~l 


> 


yO 


O 


•— * W 


o § 


1 — 


o 


en 


-< 





SUBACUTE BACTERIAL ENDOCARDITIS 



ID/CC A ."> l-Ycar-uld (finale who underwent a led mastectomy with 
axillary lymph node dissection a year ago presents with pain 
together with rapidly spreading redness and swelling of the left 
arm. 

HPI One year ago, she was diagnosed and operated on for stage 1 
carcinoma of die left breast. 

PE Left forearm swollen, indurated, pink, and markedly tender; 

overlying temperature raised: margins and holders of skin lesion 
ill defined and not elevated (vs. erysipelas). 



Labs Needle aspiration from advancing border of the lesion, when 

stained and cultured, isolated (3-hemolytic group A streptococcus. 

Treatment I'enicillinasc-rcsistaut penicillin (nafcillin/oxacillin). 

Discussion ( ielluliiis is an acute spreading infection ol the skin that 

predominantly affects deeper subcutaneous tissue. Group A 
streptococci and Staphylococcus aureus .we the most common 
etiologic agents in adults: Uai'wo/i/i/lus in/lumz/w infection is 
common in children. Patients with chronic venous stasis and 
lymphedema of any cause (lymphoma, filariasis. post-regional 
lymph node dissection, radiation therapy) are predisposed: 
recently, recurrent saphe nous-vein donor-site cellulitis was 
found to be attributable to group A. C, or G streptococci. 



CELLULITIS 






ID/CC A Ki-vear-okl U'cnager presents to the outpatient clinic with a 
painful facial rash and fever. 

HPI One week ago, the paiient went on a camping trip and 

scratched liis lace on some low-lying tree branches. There is no 
medical history of diabetes, cancel, or oilier chronic conditions. 

PE VS: fever (39.0 a C); tachycardia (HR 110): BP normal. PE : 
erythematous, warm, plaque-like rash extending across cheeks 
and face bilaterally with sharp, distinct borders and facial 
swelling. 

Labs ( IB( : leukocytosis with neutrophilia. ESR elevated. 



Treatment 



Discussion 



Atlas Link 



ERYSIPELAS 



Antibiotics with sufficient coverage for penicillinase-producing 
Streptococcus nui\ Staphylococcus spp. (e.g., cephalexin): 
analgesics/antipyretics; elevate the affected part to reduce 
swelling. 

Erysipelas is an acute inflammation of the superficial layers of 
the connective tissues of the skin, usually on the face, almost 
always caused by infection with Group A Streptococcus which is 
part of normal bacterial skin flora. Risk factors include any 
breaks in the skin or lymphedema. 

mct'j-i MC-136 



o 
m 

70 

O 

i — 

o 
en 

-< 






ID/CC A 30-year-old slaughterhouse worker presents with a painful red 
swelling of the index finger of his right hand. 

HPI The swelling developed 4 days alter he was injured with a knife 
while slaughtering a pig. 

PE Well-defined, exquisitely tender, slightly elevated violaceous 
lesion seen on right index finger; no suppuration noted; right 
epitrochlear and right axillary lynipliadenopalhv noied. 

Labs Biopsy from edge of lesion yields Erysipelothrix rhusiopathiae, 
a thin, pleomorphic, nonsporulating, microaerophilic gram- 
positive rod. 

Treatment Penicillin G or ciprofloxacin in penicillin-allergic patients. 

Discussion Erysipeloid refers to localized cellulitis, usually of the fingers 

and hands, caused by Erysipelothrix rhusiopathiae; infection in 
humans is usually the result of contact with infected animals or 
their products (often fish). Organisms gain entry via cuts and 
abrasions on the skin. 



ERYSIPELOID 



ID/CC A 10-year-old male complains of a spreading skin rash and 
painful swelling of both wrists. 

HPI The patient's mother states that die rash began with erythema 
of the cheeks ("slapped-chkkk appearance") and subsequently 
progressed to involve the trunk and limbs. 

PE Erythematous lacy/reticular skin rash involving face, trunk, and 
limbs: bilateral swelling and painful restriction of movement at 
bodi wrist joints. 

Labs Serology detects presence of specific IgM antibody to parvovirus; 
ASO titer (to rule out acute rheumatic fever) normal: 
rheumaioid factor (to rule out rheumatoid arthritis) negative. 

Treatment Self-limiting disease. 

Discussion A small (20- to 26-nm), single-stranded DNA virus, parvovirus B19 
causes erythema infectiosum (fifth disease) in schoolchildren, 
aplastic crises in persons with underlying hemolytic disorders 
(e.g., sickle cell anemia), chronic anemia in immunocompromised 
hosts, arthralgia/ arthritis in normal individuals, and fetal loss in 
pregnant women. 

Atlas Link i»icl r MC-299 



o 
m 

TO 

o 
o 

CI 

-< 



ERYTHEMA INFECTIOSUM 



ID/CC A 5 year-old whin- male presents with golden-yellow, crusted 
lesions around his mouth and behind his ears. 

HPI He has a history of intermittent low-grade lever, frequent "nose 
picking." and purulent discharge from his lesions. Me lias no 
history of hematuria (due to increased risk of poststreptococcal 
glomerulonephritis). 

PE Characteristic "honey-colored" crusted lesions seen at angle of 
mouth, around nasal orifices, and behind ears. 

Labs Gram-positive cocci in chains (si REPTOCOCCt) in addition to pus 
cells on Gram stain of discharge; (i-hciuolwic streptococci 
(group A streptococci) on blood agar culture: ASO liter negative. 

Gross Pathology Krvthematous lesions surrounding natural orifices with whitish 
or yellowish purulent exudate and crust formation. 

Micro Pathology Inflammatory infiltrate of I'MNs with varying degrees of necrosis. 

Treatment Cephalosporin, penicillin, or erythromycin if allergic. 

Discussion Impetigo is a highly communicable infectious disease that is 

most often caused bv group A streptococci, occurs primarily in 
preschoolers, and may predispose to glomerulonephritis. It 
occurs most commonly on the face (periorbital area), hands, 
and arms. Staphylococcus aureus ma) coexist or cause bullous 
impetigo: group !'■ streptococcal impetigo may be seen in 
newborns' 



IMPETIGO 



ID/CC A 30-year-old male homosexual visits his family doctor 

complaining of a nonpruritic skin eruption on his upper limbs, 
trunk, and anogenital area. 

HPI He has been HIV positive for about 3 years and admits to 
having continued unprotected intercourse. 

PE Multiple painless, pearly-white, dome-shaped, waxy, umbilicated 
nodules 2 to 5 mm in diameter on arms, trunk, and anogenital 
area; palms and soles spared. 

Gross Pathology Firm, umbilicated nodules containing thick yellowish material. 



Micro Pathology 



Treatment 



Discussion 



Atlas Link 



Stained histologic sections confirm diagnosis with large 
cytoplasmic inclusions (mou.l'scim bodies) in materia] 

expressed from lesions. 

Lesions may resolve spontaneously or be removed by curettage. 
cryotherapy, or podophyllin; no antiviral drug or vaccine 
available. 

Molluscum contagiosum is a benign, autoinoculable skin disease 
of children ;\\t(\ young adults; ii is caused b\ a poxvirus (DNA 
virus) and is transmitted through sexual contact, close bodily 
contact, clothing, or towels. It is one of many opportunistic 
infections seen in AIDS patients (difficult to eradicate). 

MC-143 



o 
m 

1 

O 
CD 

-< 



MOLLUSCUM CONTAGIOSUM 






ID/CC A 30-year-old black male presents with a nonpruritic skin rash 
on the trunk, upper arm, and neck. 

HPI The patient is otherwise in excellent health. 

PE Multiple hypopigmented, scaling, confluent macules seen on 

trunk, upper arms, and neck; no sensory loss demonstrated over 
areas of hypopigmentation; Wood's lamp examination of skin 
macules displays a pale yellow to blue-white fluorescence. 

Labs Examination of KOH mounting of scales from lesions 

demonstrates the presence of short, thick, tangled hyphae with 
clusters of large, spherical budding yeast cells with characteristic 
"spaghetti-and-meatballs" appearance. 

Treatment Topical selenium sulfide: antifungal agents such as miconazole 
and clotrimazole; oral itraconazole in recalcitrant cases. 

Discussion Pityriasis versicolor, which is common in young adults, is a 

relatively asymptomatic superficial skin infection caused by the 
lipophilic fungal organism Pityrosporum orbiculare (also 
termed Malassezia furfur) . The lesions, which usually have a 
follicular origin, are small, hypopigmented-to-tan macules with a 
branlike scale; the macules are distributed predominantly on 
areas of the upper trunk, neck, and shoulders. 

Atlas Links LTZIZLJ M-M 1-012 IT~m MC-148 



PITYRIASIS VERSICOLOR 



ID/CC An 18-month-old male is brought to the pediatrician following 
the appearance of an extensive skin rash. 

HPI Four days ago he suddenly developed a very high fever (4()°C) 

with no other symptoms or signs. The fever continued for 4 days 
until the day of his admission, when it abruptly disappeared, 
coinciding with the onset of the rash. 

PE Child looks well; in no acute distress; generalized rash apparent as 
discrete 2- to 5-mm rose-pink macules and papules on trunk, 
neck, and extremities (face is spared): lesions blanch on pressure; 
no lymphadenopaUiy; splenomegaly may also be present. 

Labs CBC/PBS: WBCs variable; relative lymphocytosis with atypical 
lymphocytes. 

Treatment Supportive; foscarnet. 

Discussion Roseola infantum, also called exanthem subitum, is caused by 
human herpesvirus 6. It is the most common exanthematous 
disease in infants 2 years of age or younger and is a frequent 
cause of febrile convulsions. 



o 

m 

TO 

o 
o 

■< 



ROSEOLA INFANTUM 






ID/CC A 2-month-old female infant presents with extensive bullae and 

large areas of denuded skin. 

HPI I ler mother had suffered from staphylococcal mastitis 1 week 
ago. 

PE VS: fever. PE: large areas of red. painful, denuded skin on 
periorbital and peribuccal areas; flaccid bullae with easy 
dislodgment of epidermis under pressure (Nikolsky's sk ;n ) ; 
mucosal surfaces largely uninvolved. 

Labs Vesicle Quid sterile; Staphylococcus aureus on blood culture. 

Treatment IV penicillinase-resistant penicillin (e.g., nafcillin, oxacillin). 

I Veal with erythromycin Jl palieni is allergic lo penicillin. 

Discussion Scalded skin syndrome is caused by the exfoliating effect of 
staphylococcal exotoxin. The action of the exoioxin is to 
degrade desmoglein in desmosomes in the skin. 






SCALDED SKIN SYNDROME 






ID/CC A 30-year-old man presents with a bilateral red pruritic skin 
eruption in the groin area. 

PE Bilateral, circular papulosquamous skin eruption on erythematous 
base with active, advancing peripheral (serpiginous) border over 
scrotum and perineum. 

Labs Microscopic examination reveals long septate hyphae on KOH 

skin scrapings. 

Treatment Topical antifungal agents (Whitfield's ointment, clotrimazole. 

miconazole); systemic therapy with oral griseofulvin, ketoconazole, 
or itraconazole in resistant cases. 

Discussion Tinea cruris and tinea corporis (common ringworm) occur 

sporadically; Trichophyton ruhrum is the most common cause. The 
inflammatory form, which is usually localized to the limbs, 
chest, or back, is commonly caused by Microsporum canis or 
Trichophyton mentagrophytes. Ringworm of the scalp, known as 
tinea capitis, is commonly seen in children and is caused by 
Trichophyton tonsurans. 

Atlas Link mcmzi MC-151 



o 

m 

1 

O 

CD 

< 



TINEA CORPORIS (RINGWORM) 






ID/CC A 28-year-old male presents with a red, pruritic skin eruption on 
his Crunk and his upper and lower limbs of a few hours' 
duration. 

HPI One day earlier, he was prescribed cotrimoxazole for a UT1. He 

has not experienced any dyspnea. 

PE Erythematous, warm, urticarial wheals (hives) seen over trunk, 
legs, and arms; no angioedema or respiratory distress. 

Labs CBC: leukocytosis with eosinophilia. No parasites revealed on 
stool exam. 



Gross Pathology 



Micro Pathology 



Treatment 



Discussion 



Linear or oval, raised papules or plaque-like wheals up to several 
centimeters in diameter. 

Wide separation of dermal collagen fibers with dilatation Ol 
lymphatics and venules. 

Topical agents to reduce itching; avoidance of causative agent 
(in this case, cotrimoxazole); antihistamines (primarily H| 
blockers but also H 2 blockers) ; consider glucocorticoids. 

Mast cells and basophils are focal to urticarial reaction. When 
stimulated by certain immunologic or nonimmunologic 
mechanisms, storage granules in these cells release histamine 
and other mediators, such as kinins and leukotrienes. These 
agents produce the localized vasodilatation and transudation of 
fluid thai characterize urticaria. 



Atlas Link IZIIO _J MC-021 



URTICARIA 



ID/CC A 7-year-old male is brought to his family physician complaining 
of a thick yellowish discharge in his eyes that prevents him from 
opening his eyes in the morning: for the past lew (lavs, his eyes 
have been blood-red, painful, and watery. His eye pain is 
exacerbated b\ exposure to light (PHOTOPHOBIA). 

HPI Three of his classmates and a neighbor had a similar episode 
about 7 days ago (suggesting a local epidemic of such cases). 

PE VS: no fever. PE: normal visual acuity; erythematous palpebral 
conjunctiva: watery eyes: remains of thick mucus found on inner 
canthal area; no corneal infiltrate on slit-lamp exam; normal 
anterior chamber; mild preauricular lymphadenopathy. 

Labs Stained conjunctival smears reveal lymphocytes, giant cells, 
neutrophils, and bacteria. 

Treatment Topical antimicrobial eye drops; cool compresses; minimize 

contact with others to avoid spread; avoid use of topical steroid 
preparations, as these can exacerbate bacterial and viral eye 

infections. 

Discussion Conjunctivitis is a common disease of childhood that is mostly 
viral (adenovirus) and self-limiting; it occurs in epidemics, and 
secondary bacterial infections (staphylococci and streptococci) 
may result. Visual acuity is not affected. 



O 



o 

■< 






ACUTE CONJUNCTIVITIS 









ID/CC A 3f>-year-old woman complains of fever and pain in the face 
and upper teeth (maxillary sinus), especially while leaning 
forward. 



HPI She has had a chronic cough, nasal congestion, and discharge 
for the pasi lew months. 

PE VS: fever. PE: halitosis: greenish-yellow postnasal discharge: 
bilateral boggy nasal mucosa; bilateral percussion tenderness 
and erythema over zygomatic arch; clouding of sinuses by 
transillumination; denial and cranial nerve exams normal. 

Labs Nasal cultures reveal Streptococcal pneumoniae. 

Imaging CT, sinus: partial opacification of maxillary sinus with air-fluid 
level. 

Gross Pathology Erythematous and edematous nasal mucosa. 

Micro Pathology Presence of organisms and leukocytes in mucosa. 

Treatment Oral decongestants; amoxicillin, Bactrim, or fluoroquinolone. 

Discussion Other pathogens include other streptococci, Haemophilus 

influenzae, and Moraxella. The obstruction of ostia in the anterior 
ethmoid and middle tneatal complex by retained secretions, 
mucosal edema, or polyps promotes sinusitis. Staphylococcus 
aureus and gram-negative species may cause chronic sinusitis. 
Fungal sinusitis may mimic chronic bacterial sinusitis. 
Complications include orbital cellulitis and abscesses. 






u 



ACUTE SINUSITIS 



ID/CC A 1 7-year-old boy presents with itchy eyes, nasal stuffiness, 

increased lacrimation, sneezing, and a watery nasal discharge. 

HPI He has had similar episodes in tin- pasi that have corresponded 
with changing of the seasons. His mother is known to have 
bronchial asthma. 

PE VS: no fever. PE: pallor; boggy nasal mucosa; nasal polyps present: 
conjunctiva congested; no exudate. 

Labs Conjunctival and nasal smear demonstrates presence of 

eosinophils; no bacteria on Gram stain; no neutrophils. Allergen 
skin tests (sensitized cutaneous mast cells) show positive 
sensitivity. 

Gross Pathology Nasal mucosa hyperemic and swollen with fluid transudation. 



Micro Pathology 



Treatment 



Discussion 



Local tissue inflammation and dysfunction of upper airway 
because of type I. IgE-mediated hypersensitivity response. 

Oral decongestants with intranasal corticosteroids; 
antihistamines: intranasal cromolyn sodium, especially before 
anticipated contact with allergen. 

Allergic rhinitis is commonly caused by exposure to pollens, 
dust content, and insect matter; symptoms arc mediated by the 
release of vasoactive and chemotactic mediators from mast cells 
and basophils (e.g., histamine and leukotrienes) with IgE 
surface receptors. 



O 
"a 



o 

-< 



ALLERGIC RHINITIS (HAY FEVER) 









ID/CC A 20-year-old male presents with a runny nose, nasal congestion, 
sore throat, headache, and sneezing. 

HPI He notes that his wife currently has similar symptoms. 

PE VS: mild fever. PE: rhinorrhea; congested and inflamed 
posterior pharyngeal wall; no lymphadenopathy. 

Labs Routine tests normal; routine throat swah staining and culture 
negative for bacteria. 

Gross Pathology Nasal membranes edematous and erythematous with watery 
discharge. 

Micro Pathology Mononuclear inflammation of mucosa; focal desquamation. 

Treatment Symptomatic. 

Discussion Colds occur 2 to 3 times a year in the average person in the 
United Slates; ihe peak incidence is in the winter months. 
Rhinoviruses account for the majority of viral URIs, followed by 
coronaviruses. Spread occurs by direct contact and respiratory 
droplets. 



COMMON COLD (VIRAL RESPIRATORY INFECTION) 



ID/CC A 60-year-old male presents with swelling and a vesicular skin 
eruption on the left side of his face. 

HPI The patient reports that before the rash developed, he had 

severe radiating pain on the left side of his face. He also recalls 
having suffered an attack of chickenpox during his childhood. 



PE Unilateral vesicular rash over left forehead and nasal bridge, 
including the Up of the nose, indicating involvement of the 
nasociliary branch of the trigeminal nerve (Hutchinson \s sign); 
skin of lids red and edematous; slit-lamp examination reveals 
numerous rounded spots composed of minute white dots 
invoking epithelium and su'oma, producing a coarse subepithelial 
punctate keratitis; cornea is insensitive. 



Micro Pathology 



Vesicular skin lesions with herpesvirus inclusions that are 
intranuclear and acidophilic with a clear halo around them 
(Cowdry type A inclusion bodies); syncytial giant cells also seen. 



Treatment Acyclovir; steroids; cycloplegics. Trifluorothymidine for HSV 
keratitis. 



o 

"O 



Discussion Herpes zoster ophthalmicus is caused by the varicella zoster 

virus, which causes chickenpox as a primary infection. Zoster is 
believed to be a reactivation of the latent viral infection. In 
zoster ophthalmicus, the chief focus of reactivation is the 
trigeminal ganglion, from which the virus travels down one or 
more branches of the ophthalmic division such that its area of 
distribution is marked out by rows of vesicles or scars left by 
the vesicles. Ocular complications arise during subsidence of ihe 
rash and are generally associated with involvement of the 
nasociliary branch of the trigeminal nerve. 



o 



HERPES ZOSTER OPHTHALMICUS 






ID/CC An 18-year-old male complains of severe irritation in the left eye, 
blurred vision, excessive lacrimation, and photophobia. 

HPI He reports that he lias had similar episodes in the past diat were 
Healed with an antiviral drug. His records indicate thai he 
suffered the first attack at the age of 7. at which time his 
condition was diagnosed and treated as a severe follicular 
keratoconjunctivitis; his records also indicate a history of 
recurrent episodes of herpes labialis. 

PE Examination of left eye reveals circumcorneal congestion; 

fluorescein staining of cornea reveals infiltrates spreading in all 
directions, coalescing with each other and forming a large, 
shallow ulcer with crenated edges ("dendritic ii.cir"): cornea is 
insensitive. 

Labs HSV-1 demonstrated on immiirioftuorescent staining of 
epithelial scrapings as well as in the aqueous humor. 

Treatment Trifluridine eye drops; acyclovir has been shown to decrease 
recurrences. 

Discussion Most ocular herpetic infections are caused by HSV-1. It is also 
the primary cause of corneal blindness in the United States. 
Primary infections present as unilateral follicular conjunctivitis, 
blepharitis, or corneal epithelial opacities; recurrences may lake 
the form of keratitis (> 90% of cases are unilateral), blepharitis, 
or keratoconjunctivitis. Branching dendritic ulcers, usually 
detected by fluorescein staining, are virtually diagnostic; deep 
stromal involvement may result in scarring, corneal thinning, 
and abnormal vascularization with resulting blindness or 
rupture of the globe. 



HSV KERATITIS 









ID/CC A 20-year-old male swimmer complains of severe pain and 

itching in the right ear that is associated with a slight amount of 
yellowish (purulent) discharge. 

HPI The patient has no previous history of discharge from the ear 
and no history of associated deafness or tinnitus. 

PE Red. swollen area seen in right external auditory meatus that is 
partially obliterating the lumen: movement of tragus is 

exquisite!) painful ( i ragai sign). 

Labs Gram stain of aural swab reveals presence of gram-negative rods; 
culture isolates Pseudomonas aeruginosa. 



Gross Pathology 



Red, swollen area seen in cartilaginous part of external auditory 
meatus; when visualized, tympanic membrane is erythematous 
and moves normally with pneumatic otoscopy (vs. acute otitis 
media). 



Treatment Eardrops (either a combination of polymyxin, neomycin, and 
hydrocortisone or ofloxacin); gentle removal of debris in ear. 



O 



Discussion Otitis externa is most common in summer months and is thought 
to arise from a change in the milieu of the external auditory 
im atus b\ increased alkali/ation and excessive moisture; this leads 
to bacterial overgrowth, most commonly with gram-negative rods 
such as Pseudomonas (also causes malignant otitis extei na)and 
Proteus or fungi such as Aspergillus. 



O 
en 

-< 



OTITIS EXTERNA 






ID/CC An 18-month-old white female presents with irritability together 
with a bilateral, profuse, and foul-smelling ear discharge of 

2 months' duration. 

HPI The patient had recurrent URIs last year, bin her mother did 

not administer the complete course of antibiotics. The patient's 
mother has a history of feeding her child while lying down. 

PE Bilateral greenish-while ear discharge; perforated tympanic 

membranes in anteroinferior quadrant of both ears: diminished 
mobility of tympanic membrane on pneumatic otoscopy. 

Labs Gram-negative coccobacilli on Gram stain of discharge from 

tympanocentesis: Haemophilus influenzae seen on culture. 



Gross Pathology 



Micro Pathology 



Possible complications include ingrowth of squamous 
epithelium on upper middle ear (CHOLESTEATOMA) if long- 
standing; conductive hearing loss; mastoiditis; and brain abscess. 

Hyperemia and edema of inner ear and throat mucosa; 
hyperemia of tympanic membrane; deposition of cholesterol 
crystals in keratinized epidermoid cells in cholesteatoma. 



Treatment Keep ear dry; amoxicillin-clavulanic acid; surgical drainage for 
severe otalgia: myringoplasty. 

Discussion Otitis media is the most common pediatric bacterial infection 
and is caused by Escherichia coU, Staphylococcus aureus, and 
Klebsiella pneumoniae in neonates; in older children il is usually 
caused by pneumococcus (Streptococcus pneumoniae). II. influenzae. 
MoraxeUa catarrhaUs, and group A streptococcus. Resistant strains 
are becoming increasingly common. 



OTITIS MEDIA 






ID/CC A 6-year-old male presents with complaints of a mild sore throat 
and eye irritation. 

HPI His mother says that he has spent hours at the community 
swimming pool this summer. 

PE Mild rhinopharyngitis; bilateral conjunctival congestion with 

scanty mucoid discharge. 

Labs Viral culture of conjunctival and nasopharyngeal swab yields 
adenovirus. 

Treatment No specific treatment; self-limiting illness. 

Discussion Adenovirus infections occur most often in infants and young 
children, who acquire the virus by the respiratory or fecal-oral 
route. The most common respiratory tract syndrome in this age 
group is mild coryza with pharyngitis; in older children, these 
symptoms may be accompanied by conjunctivitis. May also cause 
hemorrhagic cystitis in children. On electron microscopy it is 
seen as a double-stranded nonenveloped DNA virus surrounded 
by a 20-faced icosahedral protein capsid from which 12 antenna- 
like Fibers or pen tons extend radially. 



O 



o 

3 






PHARYNGITIS— ADENOVIRUS 



ID/CC A 9-year-old male complains of pain during swallowing 

(odynophagia) for 2 days, accompanied by muscle aches, 
headache, and fever. 

HPI Ik- has otherwise been in good health. 

PE V'S: level - . PE: moderale erythema of pharynx; enlarged, 
erydiematous tonsils covered with white exudate; tender 
cervical adenopathy. 

Labs CBC: neutrophilic leukocytosis. Streptococcus pyogenes isolated 

On throat swab and culture. 



Gross Pathology 



Micro Pathology 



1 Ivperemia and swelling ol upper respiraton i.ra< i mucosa; cryptic 
enlargement of tonsils with purulent exudate; enlargement of 
regional lymph nodes. 

Acute inflammatory response with polymorphonuclear infiltrate, 
hyperemia and edema with pus formation: hyperplasia of 
regional lymph nodes; dilatation of sinusoids. 



Treatment Oral penicillin V. 



Discussion 



Streptococcal pharyngitis is an acme bacterial inlei lion produced 
In gram-positive cocci in chains (Streptococcus); pharyngitis is most 
commonly caused by group A streptococcus. Complications due 
to immune-mediated cross-reactivity and molecular mimicking 
may include glomerulonephritis and rheumatic fever. 



Atlas Link 



M-Ml-026 



PHARYNGITIS— STREPTOCOCCAL 






ID/CC A 30-year-old female presents to the surgical ER complaining of 
a stabbing right upper quadrant abdominal pain. 

HPI She is a prostitute who has been receiving treatment for 
gonococcal pelvic inflammatory disease. 

PE Right upper quadrant tenderness; cervical motion tenderness 
and mucopurulent cervicitis found on pelvic exam. 

Labs Cervical swab staining and culture identifies Neisseria 
gonorrhoeae. 

Imaging US: no evidence of cholecystitis. Peritoneoscopy: presence of 

"violin string" adhesions between liver capsule and peritoneum. 

Gross Pathology Adhesions noted between liver capsule and peritoneum. 

Treatment Antibiotic therapy (ceftriaxone and doxycycline) for paiieni 
(and for partner if warranted). 

Discussion Acute fibrinous perihepatitis (Fi r/.-Hicii-Ci ktis syndrome) 
occurs as a complication of gonococcal and chlamydial pelvic 
inflammatory disease and clinically mimics cholecystitis. 



> 

—\ 

73 

O 



73 

o 

I — 

o 

CD 
-< 



FITZ-HUGH-CURTIS SYNDROME 






ID/CC A 23-year-old male presents with sudden-onset, severe vomiting, 
nausea, abdominal cramps, and diarrhea. 

HPI He had returned home about 2 hours after attending a birthday 
part) at which meat and milk were served in various forms. The 
friend who was celebrating his birthday reported similar 
symptoms. 

PE VS: no fever. PE: mild dehydration; diffuse abdominal 
tenderness; increased bowel sounds. 

Labs Toxigenic staphylococcus recovered from culturing food. 

( loagulase-positive staphylococcus cultured from nose of one of 
the cooks at party. 

Micro Pathology No mucosal lesions. 

Treatment Fluid and electrolyte balance; antibiotics not indicated. 

Discussion Staphylococcus /uncus food poisoning results from die ingestion 
of food containing preformed heat-stable enterotoxin B. 
Outbreaks of staphylococcal food poisoning occur when food 
handlers who have contaminated superficial wounds or who are 
shedding infected nasal droplets inoculate foods such as meat, 
dairy products, salad dressings, cream sauces, and custard-filled 
pastries. The incubation period ranges from 2 to 8 hours; the 
disease is self-limited. 



GASTROENTERITIS— STAPHYLOCOCCUS AUREUS 



ID/CC An 11-year-old white male presents with jaundice and dark 
yellow urine that has been present lor the last several days. 

HPI He also complains of nausea, vomiting, and malaise. For the 

past 2 weeks, he has had a low-grade level and mild abdominal 
pain. He recently returned from a vacation in Mexico, where he 
said he consumed a lot of shellfish. 

PE Icterus; tender, firm hepatomegaly; no evidence of splenomegaly 
or free fluid in the peritoneal cavity. 

Labs Direct hyperbilirubinemia; elevated serum transaminases 
(ALT > AST); moderately elevated alkaline phosphatase; 
prolonged PT; increased urinary urobilinogen and bilirubin; 
positive IgM antibody to hepatitis A (HAV) indicative of active 
HAV infection. 

Gross Pathology May often appear normal. 



Micro Pathology 



Multifocal hepatocellular necrosis with Councilman bodies; 
lymphocytic infiltrates around necrodc foci; loss of lobular 
architecture. 



Treatment Supportive management; passive vaccinadon available. 

Discussion In hepatitis A infection, virus is shed 14 to 21 days before the 
onset of jaundice; patients are no longer infectious 7 days after 
tlu onset of jaundice. It is spread by fecal-oral transmission and is 
endemic in areas where there are contaminated water sources. 
There is no chronic carrier state; recovery takes place in 6 to 
12 months. HAV is a naked, single-stranded RNA virus of the 
picorna family. A killed vaccine is available; passive immunization 
in the form of immune serum globulins is also available. 



en 

3> 

—i 
o 



o 

r— 
O 

en 
-< 



HEPATITIS A 









ID/CC A 25-year-old male medical student presents will) jaundice and 
dark yellow urine. 

HPI He admits to having experienced an accidental needle stick 
2 months ago, which he did not report. He also complains of 
nausea, low-grade fever, and loss of appetite. 

PE Icterus; tender, firm hepatomegaly: no evidence of ascites or 
splenomegaly. 

Labs Direct hyperbilirubinemia: elevated serum transaminases 

(ALT > AST): mildly elevated alkaline phosphatase: HBsAg 
posidve; IgM anti-HBc positive (present during window period). 

Imaging US, abdomen: hepatomegaly: increased echogenicity. 



Gross Pathology 



Micro Pathology 



Treatment 



Discussion 



Atlas Links 



Liver may be enlarged, congested, or jaundiced: in fulminant 
cases of massive hepatic necrosis, liver becomes small, shrunken, 
and soft (acute yellow atrophy). 

Liver biopsv reveals hepatocellular necrosis with Councilman 
bodies and ballooning degeneration; inflammation of portal 
areas with infiltration of mononuclear cells (small lymphocytes, 
plasma cells, eosinophils); prominence of Kupffer cells and bile 
ducts; cholestasis with bile plugs. 

Supportive care: follow up to determine continued presence of 
HBsAg for at least 6 months as sign of chronic hepatitis: vaccine 
available for prevention. 

Hepatitis B immune globulin plus hepatitis B vaccine are 

recommended for parenteral or mucosal exposure to blood and 
for newborns of HBsAg-positive mothers. The infection is 
divided into the prodromal, icteric, and convalescent phases; 
5% proceed to chronic hepatitis with increased risk for cirrhosis 
and hepatocellular carcinoma. Unlike hepatitis A. hepatitis B 
has a long incubation period (3 months). Hepatitis B virus is an 
enveloped, partially circular DNA virus of the hepadna family 
that contains a DNA-dependent DNA polymerase. The 
continued presence of HBsAg after infection has clinically 
resolved indicates a chronic carrier slate. 

M-Ml-030 



HEPATITIS B— ACUTE 






ID/CC A 30-year-old male is referred for an evaluation of intermittent 
jaundice over the past 2 years. 

HPI He also complains of diarrhea, skin rash, and weight loss. He 

received a blood transfusion 3 years ago, when he was injured in 
a motorcycle accident. He denies any IV drug use or any history 
of neuropsychiairic disorders in his family. 

PE Icterus: firm, tender hepatomegaly: splenomegaly: no evidence of 
ascites; no Kayser— Fleischer rings found on slit-lamp examination 
(vs. Wilson's disease). 



Labs 



Micro Pathology 



Treatment 
Discussion 



Atlas Link 



Dii eci hyperbilirubinemia; markedly raised serum transaminase 
levels; hepatitis B (HBV) serology negative; enzyme immunoassay 
of antibodies to structural and nonstructural enzyme proteins of 
hepatitis C (C200, C33c, C22-3) positive. 

On liver biopsy, presence of ballooning degeneration; fatty 
changes; portal inflammation with necrosis of hepatocytes 
within parenchyma or immediately adjacent to portal areas 

( "I'll < :l UKAI NECROSIS") . 

Ribavirin and a 2 |,-mterferon; supportive management. 

Hepatitis C belongs to the flavivirus family and is currently the 
most important cause of post-transfusion viral hepatitis: 90% of 
cases involve percutaneous transmission. Greater than 50% of 
cases progress to chronic hepatitis, leading to cirrhosis in 20%. 



M-Ml-031 



HEPATITIS C— CHRONIC ACTIVE 






ID/CC A 1 0-year-old male complains of generalized weakness, faintness 
on exertion, and occasional epigastric pain. 

HPI His mother has noticed that he often eats soil and other 
inedible things (i'K \). 

PE Pallor; puffy face and dependent edema. 

Labs CBC: microcytic, hypochromic anemia; eosinophilia. Low serum 
iron and ferritin; elevated serum transferrin; reduced bone 
marrow hemosiderin; hypoproteinemia: stool exam revealed 
eggs of Ancylosloma duodenale (ovoid eggs with thin 
transparent shell that reveal the segmented embryo within). 

Treatment Albendazole or mebendazole; iron supplementation to treat iron 
deficiency anemia. 



Discussion 



Infection with hookworms, either Ancylostoma duodenale or 
Necator americanus, is more likely where insanitary conditions 
exist: individuals at risk include children, gardeners, plumbers 
or electricians who are in contact with soil, and armed-forces 
personnel. Hookworm eggs excreted in the feces hatch in the 
soil, releasing larvae that develop into infective larvae. 
Percutaneous larval penetration is the principal mode of human 
infection. From the skin, hookworm larvae travel via the 
bloodstream to the lungs, enter die alveoli, ascend the 
bronc hotracheal tree to the pharynx, and are swallowed. Although 
transpulmonary larval passage may elicit a transient eosinophilic 
pneumonitis (Lomi i:r's pm-i momiis). this phenomenon is much 
less common with hookworm infections than with roundworm 
infections. The major health impact of hookworm infection, 
however, is iron loss resulting from the 0.1 to 0.4 mL of blood 
ingested daily by each adult worm. In malnourished hosts, such 
blood loss can lead to severe iron deficiency anemia. 






HOOKWORM 



ID/CC A 14-year-old malnourished child died soon after hospitalization 
dui' Id in extensive small bowel rupture and shock. 

HPI He had presented to the emergency room with massive bloody 
diarrhea. His history at admission revealed the presence of 
abdominal pain, fever, and diarrhea of a few days' duration; his 
symptoms had developed after he ate leftover meat at a 

fast-food restaurant. 

PE He was dehydrated, pale, and hypotensive at lime of admission 
and developed signs of peritonitis and shock shorllv before his 
death. 

Labs Culture and exam of necrotizing intestinal lesions isolated 
Clostridium perfringens type C producing beta toxin. 

Gross Pathology Autopsy revealed ruptured small intestine, mucosal ulcerations, 
and gas production in the wall. 

Micro Pathology Microscopic exam revealed necrosis and acute inflammation in 

the ileum. 



Treatment 



Discussion 



Patient died despite aggressive fluid and electrolyte replacement, 
bowel decompression, and antibiotic therapy (penicillin, 
clindamycin, or doxyevi line): surgery had been planned in view 
of rupture ol the small bowel. 

Necrotizing enterocolitis is a condition affecting poorly nourished 
persons who suddenly feast on meat (pigbel). It is associated 
with Clostridium perfringens type C and beta enterotoxin; beta 
toxin paralyzes the villi and causes friability and necrosis of the 
bowel wall. Immunization of children in New Guinea with 
beta-toxoid vaccine has dramatically decreased the incidence of 
the disease. 



Atlas Links [ZLCIZ1 M-M1-033A, M-M1-033B, PG-M1-033 



CD 

on 

—I 

O 



o 

o 

en 

-< 






NECROTIZING ENTEROCOLITIS 



ID/CC A 7-year-old male who has been hospitalized for treatment of 
acute lymphocytic leukemia complains of copious watery 
diarrhea, right lower quadrant abdominal pain, and fever. 

HPI He was diagnosed as neutropenic (due to aggressive cytotoxic 

chemotherapy ) a few days ago. 

PE VS: fever; tachycardia; tachypnea. PE: pallor; sternal tenderness; 
axillary lymphadcnopathy; hepatosplenomegaly; abdominal 
distention; moderate dehydration. 

Labs CBC: severe neuu-openia; anemia; thrombocytopenia. PBS and 
bone marrow studies suggest he is in remission: blood culture 
grows Clostridium seplicum. 

Imaging CT, abdomen; thickening of cecal wall. 

Gross Pathology Mucosal ulcers and inflammation in ileocecal region of small 
intestine. 

Treatment Aggressive supportive measures; surgical intervention; appropriate 
antibiotics (penicillin G. ampicillin. or clindamycin). 

Discussion Neutropenic enterocolitis is a fulminant form of necrotizing 

enteritis that occurs in neutropenic patients: neutropenia is often 
related to cyclic neutropenia, leukemia, aplastic anemia, or 
chemotherapy. In postmortem exams of patients who have died 
of leukemia, infections of the cecal area (typhlitis) are 
frequently found; Clostridium septicwm is the most common 
organism isolated from the blood of such patients. 






NEUTROPENIC ENTEROCOLITIS 



ID/CC A 25-year-old male complains of midepigastric pain thai usually 
brains I to 2 hours after eating and nci asionalh awakens him ai 
night, 

HPI The paiiem has heen diagnosed with duodenal ulcers several 
limes in die past, but his symptoms have consistently recurred 
even after therapy with H a blockers, antacids, and sucralfate. 

PE \'S: stable. PE: pallor; epigastric tenderness on deep palpation. 

Labs CBC: normocytic, normochromic anemia. Stool positive for 
occult blood. 

Imaging IK. II: ulcerations in annum ol stomach and duodenum: antral 
biopsy specimens yield positive urease test. 



Gross Pathology 



Micro Pathology 



Treatment 



Discussion 



Atlas Links 



Grossly round ulcer (may also be oval) seen as sharply 
punched-out defect widi relatively straight walls and slight 
overhanging ol mucosal margin (heaped-up margin is 
characteristic of a malignant lesion); smooth and clean ulcer base. 

No evidence of malignancy; antral biopsies reveal presence of 
chronic mucosal inflammation. 

Triple iherap\ with amoxicillin, metronidazole, and bismuth 
subsalicylate; triple therapy with clarithromycin, omeprazole, 
and imidazole is now considered effective and relatively free of 
side effects. 

Helicobacter pylori grows overlying the antral gastric mucosal 
cells; 40% of health) individuals and approximately 50% of 
patients with peptic disease harbor this organism. Although 
H. pylori does not breach die epithelial barrier, colonization of 
the antral mucosal layer by this organism is associated with 
structural alterations of the gastric mucosa and hence with a 
high prevalence of antral gastritis. Despite the fact that //. pylori 
docs not grow on duodenal mucosa, ii is strongly associated with 
duodenal ulcer, and eradication of the organism in patients with 
refractory peptic ulcer disease decreases the risk of recurrence. 

M-M1-035A, M-M1-035B, M-M1-035C, PG-M1-035 



> 

— t 

O 



O 

I — 

o 

cr, 

-i 






PEPTIC ULCER DISEASE (H. PYLORI) 






ID/CC A 4-year-old male is brought to the physician by his parents, who 
complain that the child has had intense perianal itching, 
especially during the night. 

HPI The child is otherwise healthy, and his developmental progress 
is normal. 

PE Perianal excoriation noted. 

Labs Cellulose adhesive tape secured to perianal area during the 
nighi reveals presence of Enterobius vermicularis eggs that 
were flattened on one side, were embryonated, and had a thick 
shell; no parasites found on stool exam. 

Treatment Strict personal hygiene; drugs used include albendazole, 
mebendazole, piperazine, and pyrantel pamoate. 

Discussion Infection is caused by Enterobius vermicularis. Adult worms are 
located primarily in the cecal region: female adult worms 
migrate to the perianal area during the night and deposit their 
eggs. Direct person-to-person infection occurs by ingestion and 
swallowing of eggs; autoinoculation occurs by contamination of 
fingers. The life cycle is completed in about 6 weeks. 



PINWORM INFECTION 






ID/CC A 10-month-old male presents with fever and severe vomiting 
followed by watery diarrhea. 

HPI His stools are loose and watery without blood or mucus. 



PE VS: fever; tachycardia. PE: child is irritable; moderate 
dehydration. 

Labs Absence of leukocytes on fecal stain: rotavirus delected with 
ELISA; electron microscopy with negative staining identifies 
rotavirus on stool ultrafiltrates. 



Micro Pathology 



Major histopathologic lesions are characterized by reversible 
involvement of the proximal small intestine; mucosa remains 
intact with shortening of villi, a mixed inflammatory infiltration 
of lamina propria, and hyperplasia of the mucosal crypt cells; 
electron microscopy reveals distended cisterns of endoplasmic 
reticulum, mitochondrial swelling, and sparse, irregular 
microvilli. 



Treatment Fluid replacement therapy. 

Discussion Rotavirus group A is the single most important cause of 

endemic, severe diarrheal illness in infants and young children 
worldwide; it occurs with greater frequency during winter 
months in temperate climates and during the dry season in 
tropical climates. In die United States, rotavirus accounts for 
50% of all childhood diarrheas, has an incubation period of 
48 hours, is transmitted by the fecal-oral route, and lasts only a 
few days. Some children subsequently develop lactose intolerance, 
which lasts for a few weeks. 



> 

— I 

73 
O 



73 
O 

I — 
O 

CD 
-< 



ROTAVIRUS DIARRHEA IN INFANTS 



ID/CC A 30-year-old male presents with sudden-onset, crampy 
abdominal pain and diarrhea. 

HPI The diarrhea is watery and contains mucus. The patient also 
Complains of low-grade lever with chills, malaise, nausea, and 
vomiting. ( iareful history reveals that he had ingested partially 
cooked eggs at a poultry I'arm 2 1 hours before his symptoms 
began. 

PE VS: fever; tachycardia. PE: mild diffuse abdominal tenderness; 
mild dehydration. 

Labs Stool culture yields Salmonella typhimurium; stained stool 
demonstrates PMNs. 

Gross Pathology Intestinal mucosal erythema (limited to the colon) and some 
superficial ulcers. 

Micro Pathology Mixed inflammatory infiltrate in mucosa; superficial epithelial 

erosions. 

Treatment Fluid and electrolyte replacement therapy; antibiotics withheld, 
as they prolong carrier state. Antibiotic therapy only for 
malnourished, severely ill, bactcrcmic, and sickle cell disease 
patients. 

Discussion Salmonella infection is acquired through the ingestion of food 
(eggs, meat, poultry) or water contaminated with animal or 
human feces; individuals with low gastric acidity are also 
susceptible. 






SALMONELLA FOOD POISONING 






ID/CC A 50-vear-old alcoholic white male presents with fever, abdominal 
pain, and rapicllv progressive disieiiiion of the abdomen. 

HPI He was diagnosed with alcoholic cirrhosis 1 month ago. when 

he was admitted to the hospital wilh jaundice and hematemesis. 

PE VS: fever. PE: icterus; on palpation, abdominal lenderness with 
guarding; fluid thrill and shifting dullness to percussion (clue to 
ascites): splenomegaly; decreased bowel sounds. 

Labs CBC: leukocytosis. Ascitic fluid leukocyte count > 500/ cc; 

PMNs (350/cc) elevated; ascitic proieins and glucose depressed: 
gram-negative bacilli in ascitic fluid; Escherichia coli isolated in 
culture; elevated AST and ALT (AST > ALT). 

Imaging KUB: ground-glass haziness (due to ascites); no evidence of free 
air. US, abdomen: cirrhotic shrunken liver; ascites; 
splenomegaly; increased portal vein diameter and flow. EGD: 
esophageal varices. 

Gross Pathology Fibrinopurulcnt exudate covering surface of peritoneum; 
fibrosis may lead to formation of adhesions. 

Micro Pathology PMNs and fibrin on serosal surfaces in various stages with 
presence of granulation tissue and fibrosis. 

Treatment Specific organism-sensitive antibiotics or empiric therapy (such 
as cefotaxime or B-lactamase-resislant penicillin) for 
gram-negative aerobic bacilli and gram-positive cocci: supportive 
treatment for cii i hosis. 

Discussion The spontaneous or primary form of peritonitis occurs in 

patients with advanced chronic liver disease and concomitant 
ascites; /.. coliis the most common cause of secondary peritonitis. 



> 

— I' 

TO 
O 



TO 

o 

I — 
o 

CD 

-< 



SPONTANEOUS BACTERIAL PERITONITIS 



ID/CC A 25-year-old male U.S. citizen on vacation in Mexico presents 
with abrupt-onset explosive watery diarrhea, abdominal cramps, 
and a low-grade fever and chills. 

HPI The patient does not complain of tenesmus or passage of blood 
or mucus in his stools, but he does complain of a feeling of 
urgency to defecate. 

PE VS: low-grade fever. PE: unremarkable. 

Labs No erythrocytes. VVBCs. or parasites seen in stained stool; 

bioassays for enterotoxigenic Escherichia coli (ETEC) reveal 
presence of the labile enterotoxin (LT) (tests available only for 
research purposes) . 

Treatment Fluid replacement; antibiotics (fluoroquinolone or TMP-SMX) 
with loperamide; prevention with careful hygienic practices and 
prophylactic fluoroquinolone or bismuth subsalicylate with 
loperamide. 

Discussion Traveler's diarrhea is a self-limited condition that develops 

within 1 to 2 days of ingestion of contaminated food or drinks. 
Over three-fourths of cases of traveler's diarrhea are caused by 
bacteria, with enterotoxigenic E. coli the most frequent cause 
(may also be caused by enteropathogenic /'.'. coli and, in Mexico, 
by an enteroadherent E. coli ). Other common pathogens 
include Shigella species. Campylobacter jejuni. Aeromonas species. 
Plesiornonas shigelloides. Salmonella species, and noncholera 
vibrios. Rotavirus and Norwalk agent are the most common viral 
causes; C.iardia, Cryptosporidium, and. rarely, Entamoeba histolytica 
are parasitic pathogens. Enterotoxigenic E. coli produce 
enteroloxins that bind to intestinal receptors and activate 
adenyl cyclase in the intestinal cell to produce an increase in the 
level of the cyclic nucleotides cAMP (LT, labile toxin) and cGMP 
(ST, stable toxin), which markedly augments sodium, chloride, 
and water loss, diereby producing a secretory diarrhea. 






53 TRAVELER'S DIARRHEA 



ID/CC A 30-year-old male presents with sudden-onset fever, colicky 
abdominal pain, and watery diarrhea. 

HPI He had eaten raw oysters at a friend's party the day before 
(incubation period 4 hours to 4 days). 

PE VS: fever; tachycardia. PE: no dehydration: diffuse abdominal 
tenderness; increased bowel sounds. 

Labs Vibrio parahaemolyticus isolated from stool in a high-salt- 
content (halophilic vibrio) culture medium; PMNs in stool; 
Kanagawa phenomenon (beta-hemolysis on medium containing 
human blood; done as an indicator for pathogenicity) positive. 

Treatment Fluid and electrolyte balance; antibiodcs not required (since 
they do not shorten course of infection). 

Discussion Seafood is the main source of the organism. After ingestion. 
Vibrio parahaemolyticus multiplies in the gut and produces a 
diarrheal enterotoxin. 



en 
> 

—I 
;o 
O 



TO 

o 
I— 
o 
c-> 



VIBRIO PARAHAEMOLYTICUS FOOD POISONING 






ID/CC A 35-year-old male presents to the emergency room with high- 
grade fever, marked weakness, and a hemorrhagic vesiculobullous 
skin eruption. 

HPI He had just returned from deep-sea fishing in die Gulf of 

Mexico, where he had consumed large quantities of seafood. 
He has been diagnosed with chronic liver disease (due to 
hemochromatosis). 

PE VS: fever: hypotension; tachycardia. PE: icterus; vesiculobullous 
skin lesions seen on an otherwise-bronzed complexion. 

Labs Blood culture on high-salt medium (halophilic bacteria) reveals 
growth of Vibrio vulnificus; evidence of hemochromatosis 

( hyperglycemia, hyperbilirubinemia, increased serum iron). 

Treatment Ceftazidime and doxycycline, ciprofloxacin; supportive. 

Discussion Halophilic Vibrio vulnificus should be suspected and treated in any 
individual with chronic liver disease who presents with sepdeemia 
and skin lesions 1 to 3 days following seafood ingestion. 






VIBRIO VULNIFICUS FOOD POISONING 



ID/CC A 56-year-old white male complains of diarrhea and bloating for 
several months along with ankle swelling. 

HPI He also complains of memory loss, fever, arthritis in the knees 
and hands, and weight loss. 

PE VS: lever. PE: thin, gaunt male; muscle wasting; swollen, tender 
right wrist and ankle; axillary and femoral lymphadenopathy; 
ecchymoses ol < hesl and arms. 

Labs CBC/PBS: macrocytic, hypochromic anemia; hypoalbnminemia; 
increased fecal fat (steatorrhea). 

Imaging l.'GI/SBFT: nonspecilic dilatation of small bowel. 

Gross Pathology Atrophy of intestinal mucosa; inflammatory infiltrate in synovia 
of joints. 



Micro Pathology 



Small bowel biopsy reveals characteristic macrophages 
containing bacilli with PAS reagent staining: characteristic 

gram-negative actinoinycete bacilli in macrophages, l'MNs. and 
epithelial cells of lamina propria; dilated lymphatics; flattening 
of intestinal villi. 



Treatment Bactrim (TMP-SMX) or ceftriaxone for 1 year. 

Discussion Caused by infection with Tropheryma whippelir. produces 

malabsorption offal-soluble vitamins, protein, iron, folic acid. 
and vitamin B| 2 . 



CD 

> 
on 

— ( 

TO 

o 



73 
o 

o 

CD 

-< 



WHIPPLE'S DISEASE 



ID/CC A 28-year-old female complains of painful swelling of both knees 
and tender skin eruptions on both shins. 

HPI For the past 2 weeks she has also had watery diarrhea that 
developed after she consumed some raw pork. She also 
complains of low-grade fever and mild abdominal pain. 

PE VS: low-grade fever; tachycardia. PE: mild dehydration; swollen 
and warm knee joints with painful restriction of all movements 
(arthritis); multiple tender, erythematous plaques and nodules 
(ERYTHEMA NODOSUM) seen over both shins. 

Labs CBC: leukocytosis. Yersinia enterocolitica isolated from stool; 
patient is HLA-B27 positive. 



Micro Pathology 



Oval ulcers with long axis in the direction of bowel flow, similar 
to ulcers caused by typhoid fever (intestinal tubercular ulcers 
are transverse). 



Treatment Supportive; antibiotics (aminoglycosides, fluoroquinolones) 
indicated in severe infections. 

Discussion Yersinia mleroailitica is an invasive gram-negative intracellular 
pathogen that causes gastroenteritis, most frequently involving 
the distal ileum and colon (enterotoxin mediated), mesenteric 
adenitis (due to necrotizing and suppurative gut lesions) and 
ileitis (pseudoappendicitis), and sepsis; infection may trigger a 
variety of autoimmune phenomena, including erythema 
nodosum, i ca< live arthritis, and possibh < Waves' disease. 
especially in HLA-B27-positive individuals. Spread is by the 
fecal-oral route and occurs via contaminated milk products or 
water, swine, or household pet feces. 



YERSINIA ENTEROCOLITIS 






ID/CC A 3-year-old albino male is referred to a specialist for an 
evaluation of a suspected immune deficiency. 

HPI His parents report recurrent respiratory, skin, and oral 

infections with gram-negative and gram-positive organisms. He 

also has a history of bruising easily. 

PE Partial albinism; light-brown hair with silvery tint; nystagmus; 
photophobia on eye reflex exam; chronic gingivitis and 
periodontitis; purpuric patches over areas of repeated minimal 
trauma; mild hepatomegaly; no lymphadenopathy. 

Labs CBC/PBS: decreased neutrophil count with normal platelet 
count: large cytoplasmic granules (c.ian i l.ysosomes) in WBCs 
on Wright-stained peripheral blood smears. Prolonged bleeding 
time: impaired platelet aggregation; normal clotting time and 
PTT; normal nitroblue tetrazolium test. 

Treatment Largely supportive; ascorbic acid, prophylactic antibiotics, 
acyclovir. 

Discussion Chediak-Higashi syndrome is an autosomal-recessive disorder 
thai is due to a defect in polymerization of microtubules in 
leukocytes that causes impairment of cbemotaxis, phagocytosis, 
and formation of phagolysosomes. Patients with this disorder 
usually present with recurrent pyogenic staphylococcal and 
streptococcal infections. 



CI 






CHEDIAK-HIGASHI SYNDROME 






ID/CC An 8-year-old child with sickle cell anemia is seen with 

complaints of sudden-onsel pallor of the skin and mucous 
membranes, fatigue, and malaise. 

HPI The- child suffered a mild prodromal illness before developing 
severe pallor. 

PE VS: no fever; tachycardia: tachypnea; BP normal. PE: severe 
pallor; mild icterus; no lymphadenopathy, splenomegaly, or 
hepatomegaly noted. 

Labs CBC: severe anemia (Hb 2 g/dL); reduced leukocyte and 

plaielei counts: mild hvperbilirubinemia; absent reticulocytes 
and sickled RBCs on peripheral blood smear. 



Micro Pathology 



Bone marrow biopsy reveals increased numbers of giant 
pronormoblasts (diagnostic of parvovirus infection). 



Treatment Blood transfusions to tide over the crises. Spontaneous recovery 

in 1 to 2 weeks. 

Discussion Parvovirus infection is the cause of transient aplastic crises (may 

also be due to folic acid deficiency) that occur in patients who have 
severe hemolytic disorders; cessation of erythropoiesis for about 10 
days in a normal adult as a result of parvovirus infection would 
produce a 10% drop in hemoglobin concentration (i.e., a fall of 
1% dailv would lead to a decline in hemoglobin concentration of I 
to 2 g/dL after 10 days). A patient with severe hemolysis in 
whom the bone mat 1 i >w is turning over at a rale seven times 
normal would experience a 70% decrease in hemoglobin con- 
centration (i.e., a drop from 10 g/dL to 3 g/dL) as a result of a 
10-dav cessation of erythropoiesis. Although parvovirus can 
affect all precursor cells, the red cell precursors are most 
profoundly affected. 

Atlas Link □ □ 1 H-Ml-046 



ANEMIA— APLASTIC CRISIS (PARVOVIRUS B19) 



ID/CC A 35-year-old Finnish man complains of easy fatigability and 
shortness of breath. 

HPI He often cats undercooked or raw freshwater fish. He also 
reports vague digestive disturbances such as anorexia, 
heartburn, and nausea. 

PE PE: pallor. 

Labs CB( /1'BS: megaloblastic anemia. Blood vitamin B, 2 levels low; 
stool exam reveals presence of operculated eggs and proglottids 
of Diphyllobothrium latum. 

Treatment Niclosamide or praziquantel. 

Discussion Diphyllobothrium latum (fish tapeworm) infection is found in cold 
climates where raw or undercooked fish are eaten. The adult 
worm attaches to the human jejunum and competes for 
absorption of vitamin B, 2 . producing a deficiency that resembles 
pernicious anemia. Prevention includes proper preparation of 
fish. 



ANEMIA— DIPHYLLOBOTHRIUM LATUM 






ID/CC A 45-year-old male with refractory acute myeloid leukemia who 
underwent a bone marrow transplant from a nonidentical 
donor presents with an extensive skin rash, severe diarrhea, and 
jaundice. 

HPI Prior to the transplant, which occurred 2 months ago. he 

received preparative chemotherapy and radiotherapy along with 
broad-spectrum antibiotics. Engraftment was confirmed within 
4 weeks by rising leukocyte counts. 

PE VS: BP normal. PE: patient is cachectic and moderately 
dehydrated: icterus noted; violaceous, scaly macules and 
erythematous papules resembling lichen planus seen over 
extremities. 

Labs CBC: tailing blood counts; relative eosinophilia. Elevated direct 
serum bilirubin and transaminases: stool exam reveals no 
infectious etiology: skin biopsy taken. 



Gross Pathology 



Treatment 



Discussion 



Skin biopsy specimens reveal vacuolar changes of basal cell layer 
with perivenular lymphocytic infiltrates (CD8+ T cells). 

High-dose cyclosporine therapy, rabbit anti-thymocyte globulin, 
methylprednisolone or anti-T-ceU monoclonal antibodies. 

Approximately 30% of bone marrow transplant recipients 
develop "lali-versus-host disease- ((AUD). This attack is primarily 
launched bv immunocompetent T lymphocytes derived from 
the donor's marrow against the cells and tissues of the recipient, 
which it recognizes as foreign. Cyclosporin A is effective for 
prevention of GVHD. 



GRAFT-VERSUS-HOST DISEASE 









ID/CC A 20-year-old male presents with an extensive purpuric skin rash, 
oliguria, and marked weakness; he also complains of bloody 
diarrhea of 1 week's duration. 

HPI The patient ate a hamburger at a fast-food restaurant 2 to 3 days 
prior to the onset of his diarrhea. He has no associated fever. 

PE VS; no fever. PE: dehydration; pallor; extensive purpuric skin 
rash. 



Labs 



Imaging 



Micro Pathology 



Treatment 



Discussion 



Atlas Link 



Stool examination reveals presence of RBCs but no inflammatory 
cells or parasites; culture isolates sorbitol-negative Escherichia 
coli; serotyping studies and effect on HeLa cell culture reveal 
presence of enterohemorrhagic E. coli (EHEC) serotype 
0157:H7. elevated BUN and creatinine. CBC/PBS: microangio- 
pathic anemia and thrombocytopenia. PT, PIT normal. 

Sigmoidoscopy: moderately hyperemic mucosa with no evidence 
of any ulceration. 

Pathology localized to kidney, where hyaline thrombi were seen 
in afferent arterioles and glomerular capillaries. 

Dialysis and blood transfusion for management of HUS; Quid and 
electrolyte maintenance; antimicrobial therapy. Most patients who 
develop HUS as a complication of E. coli hemorrhagic colitis die 
as a result of hemorrhagic complications. 

Hemorrhagic colitis associaied with a Shiga-like toxin producing 
EHEC 0157:H7 is characterized by grossly bloody diarrhea with 
remarkably little fever or inflammatory exudate in stool; a 
significant number of patients develop potentially fatal HUS. 
EHEC infections can be largely prevented through adequate 
cooking of beef, especially hamburgers. 



Li _.] H-Ml-049 



5 

o 
r~ 
O 
CI 

-< 



o 
I — 
O 

£71 

■< 



HEMOLYTIC-UREMIC SYNDROME (HUS) 









ID/CC A 34-year-old male presents to his primary care physician with a 
hard, red, painless swelling on his left mandible that has slowly 
been growing over die past few weeks and has now begun to 
drain pus. 

HPI The patient recendy had a tooth extracdon. 

PE No acute distress; no other significant findings. 

Labs Gram stain of exudate reveals branching gram-positive filaments 
and characteristic "sulfur granules": non-acid-fast and anaerobic 
(distinguishes actinomyces from Nocardia). 

Imaging XR: no bony destruction. 

Gross Pathology Sinus tracts from region of infection to surface with granular 
exudate. 



Micro Pathology 



Treatment 



Discussion 



Granulation tissue and fibrosis surrounding a central suppurative 
necrosis; granulation tissue may also enclose foamy histiocytes 
and plasma cells. 

Ampicillin followed by amoxicillin or penicillin G followed by 
oral penicillin V and, if necessary, surgical drainage and removal 
of necrotic tissue. 

Actinomyces israelii is a part of the normal flora of the mouth 
(crypts of tonsils and tartar of teeth), so most patients have a 
history of surgery or trauma. There is no person-to-person 
spread. Actinomycosis is a chronic suppurative infection and 
can also involve the abdomen or lungs, especially following a 
penetrating trauma such as a bullet wound or an intestinal 
perforation. Pelvic disease is associated with 1UD use. Spread 
occurs contiguously, not hcmatogenously. 



Atlas Link L'KMI 1 1 M-Ml-050 



ACTINOMYCOSIS 






ID/CC A 7-month-old girl is brought to the pediatric clinic with 

wheezing, respiratory difficulty, and nasal congestion of 3 hours' 
duration. 

HPI She has had rhinorrhea, fever, and cough and had been 
sneezing for 2 days prior to her visit to the clinic. 

PE VS: tachypnea. PR: nasal flaring; mild central cyanosis; accessory 
muscle use during respiration; hyperexpansion of chest; 
expiratory and inspiratory wheezes; rhonchi over both lung fields. 

Labs CBC/PBS: relative lymphocytosis. ABGs: hypoxemia with mild 

hypercapnia. Respiratory syncytial virus (RSV) demonstrated on 

viral culture of lluoal swab. 

Imaging CXR: hyperinflation; segmental atelectasis; interstitial infiltrates. 

Micro Pathology Severe bronchiolitis produces bronchiolar epidielial necrosis, 
lymphocytic infiltrate, and alveolar exudates. 

Treatment Humidified oxygen, bronchodilators. aerosolized ribavirin. 



o 

—I 

t—H 

o 

c= 

I — I 
CO 

m 

m 



Discussion RSV is the most common cause of bronchiolitis in infants under 
2 years of age; other viral causes include parainfluenza, 
influenza, and adenovirus. RSV shedding may last 2 or more 
weeks in children. 



m 



ACUTE BRONCHIOLITIS 






ID/CC An 8-year-old female presents with pain and swelling of her knee 
joints, elbows, and lower limbs along with fever for the past 
2 weeks; she also complains of shortness of breath (dyspnea) on 
exertion. 

HPI The patient had a sore throat 2 weeks ago. 

PE VS: fever. PE: blanching, ring-shaped erythematous rash over 
trunk and proximal extremities (erythema marc.i.vyi I'M); 
subcutaneous nodules at occiput and below extensor tendons in 
elbow; swelling with redness of both knee joints and elbows 
(POLYARTHRI i is); painfully restricted movement; pedal edema; 
increased JVP; high-frequency apical systolic murmur with 
radiation to axillae (mitral valve insufficiency due to carditis); 
bilateral fine inspiratory basal crepitant rales; mild, tender 
hepatomegaly. 

Labs CBC: leukocytosis. Streptococcus pyogenes an throat swab; markedly 
elevated ASO titers; elevated ESR; elevated C-reactive protein 
(CRP); negative blood culture. EGG: prolonged P-R interval. 



Imaging 



Gross Pathology 



Micro Pathology 



CXR: cardiomegaly; increased pulmonary vascular markings. 
Echo: vegetations over mitral valve with regurgitation. 

Acute form characterized bv endo-, myo-, and pericarditis 

( PAN( ARDrns); chronic form characterized by fibrous scarring with 

calcification and mitral stenosis with verrucous fibrin deposits. 

Myocardial muscle liber necrosis enmeshed in collagen; 
characteristic finding is fibrinoid necrosis surrounded by 
perivascular accumulation of mononuclear inflammatory cells 

(AsCllOKKCItl-S). 



Treatment Aspirin, corticosteroids, and diuretics; penicillin or 

erythromycin. 

Discussion Acute rheumatic fever is a sequela of upper respiratory infection 
with group A, p-hemolytic streptococcus; it causes autoimmune 
damage to several organs, primarily the heart. The systemic 
effects of acute rheumatic fever are immune mediated and are 
secondary to cross-reactivity of host antistreptococcal antibodies. 

Atlas Link L_1_L_LJ M-Ml-052 






ACUTE RHEUMATIC FEVER 



ID/CC A 48-year-old missionary who has lived in Cameroon, 

West Africa, for 20 years is airlifted home because of lethargy, 
nuchal rigidity, persistent headache, and drowsiness that have 
not responded to antibiotics and supportive treatment. 

HPI He states that over the years he has been bitten in the neck 
several times by a muttimutu, or tsetse fly (Gui.ss/\.\ palpaijs). 
lie has also had intermittent, generalized erythematous rashes 
accompanied by fever. 

PE Alert but somewhat incoherent and confused; sometimes 
delusional; nuchal rigidity and tremors of face and lips; 
splenomegaly; generalized rubbery, painless lymphadenopathy, 
predominantly in posterior neck and supraclavicular areas 
(Wjnterbottom's sign). 



Labs 



Gross Pathology 



Micro Pathology 



PBS/LP: hypercellular, trypanosomal forms present; 
lymphocytes in CSF. Elevated IgM. 

Chancre with erythema and induration at bite site; chancre 
resolves spontaneously; spleen and lymph nodes enlarged 
during systemic stage; leptomeninges enlarged during CNS 
involvement. 

Skin: edema, mononuclear cell inflammation, organisms, and 
endothelial proliferation; spleen and lymph nodes: histiocytic 
hyperplasia; CNS: mononuclear cell meningoencephalitis. 



Treatment Suramin; pentamidine or eflornithine. 

Discussion Also called sleeping sickness. African trypanosomiasis is a 

systemic febrile disease endemic to Africa whose chronic form 
causes a meningoencephalitis. It is caused by the flagellated 
protozoans Trypanosoma brucei gambiense (West African) and 
Trypanosoma brum rhodesiense (East African), which are transmitted 
by the tsetse fly. 



O 

c= 
1/1 

a 
i— i 
tn 
m 
> 



AFRICAN TRYPANOSOMIASIS 



ID/CC A 28-year-old male homosexual complains of continuous 

low-grade fever, weight loss, and diarrhea of 1 month's duration. 

HPI He also complains of an extensive skin rash, mucous membrane 
eruptions, recurrent herpes zoster infection, and oral 
ulcerations. He reports practicing receptive anal intercourse. 

PE VS: low-grade fever. PS: cachectic; generalized lymphadenopathy; 
maculopapular rash: severe seborrheic dermatitis; aphthous 
ulcers: while confluent patch with corrugated surface (oral. 
HAIRY LEUKOPLAKIA) along lateral borders of tongue: penile warts 

(i MMiM.nMAi \ v i mix m \ i; extensive multiple pruritic, pink. 
umbilicaied papules 2 to 5 mm in diameter (molli si i m 
contac.iosum). 

Labs CBC: anemia; leukopenia with lymphopenia; thrombocytopenia. 
Low CD4+ count; elevated CD8+ T-cell count; ELISA for HIV-1 
positive; Western blot confirmatory; PCR for viral RNA 

(investigation of choice in window period) positive. 



Micro Pathology 



Treatment 



Oral hairy leukoplakia; lesions show keratin projections 
resembling hairs, koilocytosis, and little aiypia; hybridization 
techniques reveal Epstein-Barr virus in lesions. 

Prophylactic antibiotics for prevention of opportunistic 
infections while monitoring CD4+ T-cell counts; anthetioviral 
drugs (zidovudine, didanosine, zalcitabine, and protease 
inhibitors); counseling and rehabilitative measures. 



Discussion AIDS-related complex (ARC) consists of symptomatic conditions 
in an HIV-infected patient thai are noi included in the AIDS 
surveillance case definition and that meet at least one of the 
following criteria: (1) the conditions are indicative of a defect in 
i ell-mediated immunity: oi ("_') the conditions have a ( linical 
course or management that is complicated by HIV infection. 

Atlas Link QUI Z-Ml-054 



AIDS-RELATED COMPLEX (ARC) 



ID/CC A 28-year-old male from India complains of gradual-onset, inter- 
mittent, crampy abdominal pain with one to four foul-smelling, 
frothy loose stools daily. 

HPI His stools sometimes contain blood and mucus. He also 

complains of flatulence, tenesmus, and, at times, alternating 
diarrhea and constipation. 

PE Slight tenderness during palpation of cecum and ascending 
colon; no hepatomegaly. 

Labs CBC: mild leukocytosis; no eosinophilia. Fresh stool examinadon 
reveals presence of Entamoeba histolytica cysts and motile 
hematophagous trophozoites; serology for antiamebic antibodies 
is positive. 



Imaging 



Micro Pathology 



Treatment 



Colonoscopy: multiple colonic mucosal ulcers [hat are slightly 
raised and covered with shaggy exudate; mucosa between ulcers 
normal. 

Biopsy specimens reveal lesions extending under adjacent intact 
mucosa to produce classical "flask-shaped" ulcers; amebic 
trophozoites demonstrated at base of ulcer. 

Metronidazole (drug of choice) followed by paromomycin or 
iodoquinol. 



m 

rtsi 



o 

cr 
oo 

o 

I — I . 
m 

> 



Discussion Entamoeba histolytica cysts are infective and are transmitted 

through contaminated water, raw vegetables, food handlers, and 
fecal-oral or oral-anal contact. The siles of involvement, in order 
of frequency, are the cecum and ascending colon, rectum, 
sigmoid colon, appendix, and terminal ileum. Trophozoites are 
the invasive form of the organism, causing colitis or distant 
infection by hematogenous spread. Complications include 
perforation of the bowel; liver abscess with pleural, pericardial, 
or peritoneal rupture; bowel obstruction by ameboma; and skin 
ulcers around the perineum and genitalia. 



Atlas Link 



M-Ml-055 



AMEBIC COLITIS 



ID/CC A 45-year-old male Peace Corps volunteer who recently spent 
2 years in rural Mexico complains of a spiking fever, malaise, 
headache, and right upper quadrant abdominal pain. 

HPI He admits to having had bloody diarrhea with mucus 

(nvsi-MiRY) and tenesmus dial disappeared with some pills that 
he took several months ago. 

PE VS: fever (39.6°C). PE: pallor: slight jaundice; tender 3 + 
hepatomegaly with no rebound tenderness; pain on list 
percussion of right lower ribs. 

Labs CBC: leukocytosis with neutrophilia. Amebic cysts in stool 

specimen (not concurrent with abscess); positive serology for 
antibodies to Entamoeba histolytica. 



Imaging 



Gross Pathology 



CXR: elevation of right hemidiaphragm; small right pleural 
effusion. CT/US: cavitating lesion in right lobe of liver (due to 

abscess). 

Multiple colonic mucosal ulcers, slightly raised and covered with 
shaggy exudate: enlarged liver with one large abscess on right 
lobe containing chocolate-colored pus; abscess may rupture and 
spread to lungs, brain, or other organs. 



Micro Pathology Sierile pus; ann-b.i may be obtained from periphery of lesion. 

Treatment Metronidazole; needle evacuation; surgery in case of treatment 
failure or rupture. 

Discussion Prior travel lo endemic areas plus a triad ol lever, hepatomegaly, 
and right upper quadrani pain are hallmarks of hepatic liver 
abscess. Colitis precedes the liver abscess: amebas llien invade 
the gut wall and enter portal circulation. 



AMEBIC LIVER ABSCESS 



ID/CC A 15-year-old male who resides in Florida presents with nausea 
and vomiting, fever, and marked neck stiffness. 

HPI He also complains of a severe bifrontal headache. Careful 

liisioi v reveals that he swam for several hours in brackish water 

approximately a week ago. 

PE VS: fever; tachycardia. PE: signs of meningeal irritation (neck 
rigidity, positive Kernig's sign and Brudzinski's sign): 
I'unduscopy reveals mild papilledema. 



Labs 



Gross Pathology 



Micro Pathology 



LP: bloody CSF (raised RBC count may also be due to examiner's 
inability to recognize proliferating amebas) shows intense 
neutrophilia, pleocytosis, high protein, and low sugar; no 
organism seen on Gram, ZN. or India ink staining of CSF; wet 
preparation of CSF reveals viable Naegleria trophozoites; 
diagnosis confirmed using direct fluorescein antibody staining. 

lesions are mostly present in the olfactory nerves and brain. 
Focal hemorrhages, extensive fibrinoid necrosis, and blood 
vessel thrombosis with nerve tissue necrosis. 

Naegleria fowkri trophozoites seen as 10- to 20-u.in-diameter 
oiganisms with large nucleus, small granular cytoplasm, distinct 
ectoplasm, and bulbous pseudopodia. 



Treatment Intracisternal and IV amphotericin B, miconazole, rifampin; 
prognosis is very poor. 

Discussion Primary amebic meningoencephalitis is caused by amebas of the 
genus Niu-gfcrin or Arniilliamoi'l/ri. The former most often affects 
children and voting adults, appears to be acquired by swimming 
in warm, fresh/brackish water, and is almost always fatal, with 
die ameba gaining entry into die arachnoid space through the 
nasal cribriform plate. Acantkamoeba infections involve older, 
immunocompromised individuals and are sometimes 
characterized by spontaneous recovery. 



AMEBIC MENINGOENCEPHALITIS 



o 

cz 

I — I 
on 
m 
> 

m 






ID/CC A 30-year-old male goes to the emergency room because of 
dyspnea, cyanosis, hemoptysis, and chest pain. 

HPI He lias had a high lever, malaise, and a nonproductive cough for 
1 week. The patient is a sheep farmer who remembers having 
been treated for dark black skin lesions in the past. 

PE VS: fever. PE: dyspnea; cyanosis; bilateral rales heard over lungs. 

Labs CBC: normal. Negative blood and sputum cultures; diagnosis 
of anthrax confirmed by fourfold increase in indirect 
microhemagglutination titer. 

Imaging CXR: mediastinal widening. CT, chest: evidence of 
"hemorrhagic mediastinitis." 

Gross Pathology Patchy consolidation: vesicular papules covered by black eschar. 

Micro Pathology Lungs show fibrinous exudate with many organisms but few 
PMNs. 

Treatment Isolate and treat with TV penicillin G or ciprofloxacin. 



Discussion 



Anthrax is caused by infection with Bacillus anthracis. A 
cell-free anthrax vaccine is available to protect those employed 
in industries associated with a high risk of anthrax transmission 
(farmers, veterinarians, tannery or wool workers) . 



E3 



ANTHRAX 



ID/CC A 38-year-old male receiving cytotoxic chemotherapy 

(immunosuppressed) for acute leukemia presents with pleuritic 
chest pain, hemoptysis, fever, and chills. 

HPI He also complains of dyspnea, tachypnea, and a productive 
cough, 

PE VS: fever: PE: severe respiratory distress; bilateral rales heard 
over lungs. 

Labs CBC: severe neutropenia. Negative blood and sputum culture 
for bacteria. 

Imaging CXR: necrotizing bronchopneumonia. 

Gross Pathology Necrotizing bronchopneumonia: abscesses. 

Micro Pathology Lung biopsy identifies Aspergillus -with septate, acutely branching 
hvphae (visualized by silver stains); necrotizing inflammation; 
vascular thrombi with hyphac (due to blood vessel invasion). 

Treatment IV amphotericin B or itraconazole. 

Discussion The most lethal form of infection, invasive aspergillosis, is seen 
primarily in severely immunocompromised individuals, i.e., 
patients with AIDS: patients with prolonged, severe neutropenia 
following cytotoxic chemotherapy; patients with chronic 
granulomatous disease; and patients receiving glucocorticoids 
and Other immunosuppressive drugs (e.g., transplant recipients). 

Atlas Links UTEX M-M1-059A, M-M1-059B, M-M1-059C 



o 

— i 
i — i 
o 

cz 

CO 

CT 
i — i 

oo 
m 
> 
1/1 



ASPERGILLOSIS 






ID/CC A 50-year-old male presents to the ER with complaints of 

recurrent, sudden-onset, severe breathlessness, wheezing, fever, 
chills, and a productive cough (sometimes producing brown 
bronchial casts) . 

HPI The patient has had steroid-dependent chronic bronchial 

asthma for many years and has no history of foreign travel or 
contact with a TB patient. He has a history of occasional 
hemoptysis. 

PE VS: fever; marked tachycardia; severe tachypnea. PE: respiratory 
distress; central cyanosis; wheezing: rhonchi and coarse rales 
over both lung fields. 

Labs CBC: eosinopbilia. Oxygen saturation low. Very high liters of 
specific IgE antibodies against Aspergillus present (specific 
marker for the disease); sputum cultures positive for Aspergillus; 
skin tests to Aspergillus -antigens positive. PFI's: obstructive 
picture (due to underlying asthma). 

Imaging CXR: segmental infiltrate in upper lobes (these infiltrates are 
segmental because they correspond directly to the affected 
bronchi); branching, fingerlike shadows from mucoid impaction 
of dilated central bronchi (virtually pathognomonic of allergic 
bronchopulmonary aspergillosis). CT, chest: evidence of 
proximal bronchiectasis. 

Treatment Oral corticosteroids or beclomethasone. 

Discussion Allergic bronchopulmonary aspergillosis (ABPA) is a 

hypersensitivity disorder that primarily affects the central 
bronchi: immediate and Arthus-type hypersensitivity reactions 
are involved in its pathogenesis. The onset of the disease occurs 
most often in the fourth and lillh decades, and virtually all 
patients have long-standing atopic asthma. Untreated ABPA 
leads to proximal bronchiectasis. 



ASPERGILLOSIS— ALLERGIC BRONCHOPULMONARY 









ID/CC A 50-year-old alcoholic male presents with a high-grade fever, 
cough, copious, foul-smelling sputum, and pleuritic right-sided 
chest pain. 

HPI His wife reports that he was brought home in a semiconscious 
state a few days ago, when he was found lying on the roadside 
heavilv under the influence of alcohol. 

PE VS: fever. PE: signs of consolidation elicited over right middle 
and lower pulmonary lobes. 

Labs Sputum reveals abundant PMN leukocytes and mixed oral flora; 
culture yields Bacteroides melaninogenicus (Prevotella 
melaninogenica ) and other Bacteroides species, Fusobacterium, 
microaerophilic streptococci, and Peptostreptococcus. 

Imaging ( ;XK: consolidation involving apical segment of right lower lobe 
and posterior segments of middle lobe; large cavity with air-fluid 
level (abscess) also seen. 

Treatment Clindamycin. 

Discussion Alcoholism, drug abuse, administration of sedatives or anesthesia. 
head trauma, and seizures or other neurologic disorders are 
most often responsible for the development of aspiration 
pneumonia. Because anaerobes are the dominant flora of the 
upper GI tract (outnumbering aerobic or facultative bacteria by 
10 to I ), they are the dominant organisms in aspiration pneu- 
monia; of particular importance are Bacteroides melaninogenicus 
( Prevotella melaninogenica) and other Bacteroides species (slender, 
pleomorphic, pale gram-negative rods), Fusoiia el 'mum nucleatum 
(slender gram-negative rods with pointed ends), and anaerobic 
or microaerophilic streptococci and Peptostreptococcus (small 
gram-positive cocci in chains or clumps). 



O 

cz 

I — I 
in 
m 
> 
E/i 






ASPIRATION PNEUMONIA WITH LUNG ABSCESS 






ID/CC A 38-year-old HIV-positive male is admitted to the hospital with 
fever, rigors, night sweats, and diarrhea. 

HPI He reports excessive weighi loss over the pasi few weeks. He was 
treated lor Pneumocystis pneumonia a few weeks ago and still 
reports a persistent productive cough. 

PE \'S: lexer. PE: palieni is extremely emaciated: hcpaiosplenomegaly 
and lymphadenopathy noted. 



Labs 



Imaging 



Micro Pathology 



CD4+ count < 50/cc: Mycobacterium aviutn-iiitracellulare 
isolated on blood culture; smears of tissues obtained from 
lymph nodes, bone marrow, spleen, liver, and lungs reveal 
evidence of acid-fast bacilli, and cultures yield ML avium; 
intestinal infection with M. avium proven by culture of stools 
and colonic biopsy specimens. 

CT. abdomen: hepatosplenomegaly; retroperitoneal 

lymphadenopathy; bowel mucosal fold thickening. 

Despite the presence of many mycobacteria and macrophages, 
well-formed granulomas were typically absent due to profound 
impairment of cell-mediated immunity. 



Treatment The primary treatment regimen includes clarithromycin and 

ethambutol with or without rifabutin. The failure rate of therapy 
is high. 

Discussion Mycobacterium avium complex is now the most frequent 
opportunistic bacterial infection in patients with AIDS; it 
typically occurs late in the course of the syndrome, when other 
opportunist ii infections and neoplasia have already occurred. 
Prophylaxis against M. avium-intrairltularea recommended in 
AIDS patients with a CD4+ count of < LOO/ram 3 (administer 
azithromycin, clarithromycin, or rifabutin). 



ATYPICAL MYCOBACTERIAL INFECTION 



ID/CC A 20-year-old male from India presents to the ER with severe 
nausea and vomiting. 

HPI Careful history reveals that 2 hours ago he ate some 

unrefrigerated fried rice that his wife had cooked the night 
before. He does not complain of any fever or diarrhea (may or 
may not be present) . 

PE VS: no fever. PE: mild dehydration; diffuse mild abdominal 
tenderness. 

Labs Fecal staining reveals no RBCs, WBCs, or parasites; Bacillus 

cereus, a gram-positive rod, isolated from vomitus and stool and 
shown to produce the emetogenic enterotoxin. 

Treatment Supportive. 

Discussion Bacillus cereus causes two distinct syndromes: a diarrheal form 
(mediated by an Escherichia ro/JLT-type enterotoxin witb an 
incubation period of 8 to lb hours: caused by meats and 
vegetables) and an emetic form (mediated by a Staphylococcus 
aureus-XYpe enterotoxin with an incubation period of 1 to 
8 hours; caused by fried rice). Proper food handling and 
refrigeraiion of boiled rice are largely preventive. 



s 

cz; 

I — I 
Co: 
m 
> 
CO 






BACILLUS CEREUS FOOD POISONING 






ID/CC A MO-vcar-old male who recently emigrated from Peru presents 
wiili an extensive nodular skin eruption, mild arthralgias, and 
occasional fever. 

HPI One month ago. the patient had a high-grade fever that was 
accompanied by excessive weakness, dyspnea, and passage of 
cola-colored urine; the fever subsided after '_' weeks, bill his 
weakness has progressed since that lime. 

PE Pallor; mild icterus; extensive skin rash comprising purplish 
nodular lesions of varying sizes seen on face, trunk, and limbs; 
mild hcpaiosplenomcgaly; lunduscopy reveals retinal 
hemorrhages. 



Labs 



Micro Pathology 



Treatment 



Discussion 



Intraerythrocytic coccobacillary-form bacteria \isible in thick 
and thin 61ms stained wild Giemsa; bacteria seen and isolated 
from skin lesions; indirect serum bilirubin elevated. PBS; 
macrocytic, hypochromic anemia with poh/chromasia; marked 
reticulocytosis (due io hemolytic anemia); Coombs' lest negative. 

Skin biopsy of vascular skin lesions reveals endothelial proliferation 
and histiocytic hyperplasia; electron microscopy of verrucous tissue 
sin iws Bartonella bacilliformis in interstitial tissue. 

Chloramphenicol, penicillin, erythromycin, norfloxacin, and 
tetracycline are effective; rifampicin is indicated for treatment of 
verrucous forms. 

Bartonellosis is a sandfly-borne bacterial disease occurring only 
on ihe western coast of South America at high altitudes: the 
causative agent is a motile, pleomorphic bacillus, Bartonella 
bacilliformis. Two stages of the disease are recognized: an initial 
febrile stage associated with a hemolytic anemia (Oron \ 1 1 \ 1 k) 
and a later cutaneous stage characterized by hemangiomatous 
nodules (VERRUGA PERI ana). 






BARTONELLOSIS 



ID/CC A 32-year-old male is referred to a tertiary care center with 
chronic pneumonia and warty lesions on his left upper limb. 

HPI The patient is from the southeastern United States. His skin 

Lesions are nonpruritic and painless. He also complains of 
malaise, weight loss, night sweats, chest pain, breathlessness, and 
hoarseness. 

PE VS: fever; tachycardia; mild tachypnea. PE: bilateral rales and 
rhonchi; raised, verrucous, and crusted lesions with serpiginous 
border located on left upper extremity; small abscesses 
demonstrable when superficial crust was removed. 

Labs Sputum and pus from cutaneous lesions demonstrate spherical 
cells (8 to 15 mm in diameter) that have a thick-walled, 
refracdle double contour and show unipolar (broad-based) 
budding: culture of pus and sputum on Sabouraud's agar yields 
growth of Blastomyces; no evidence ol acid-fast bacilli found 
either on staining or on culture; Gomori's methenamine silver 
staining of lung tissue does not reveal Pneumocystis. 

Imaging CXR: bilateral alveolar consolidations with air bronchogi urns. 



Micro Pathology 



Epithelioid macrophages and giant cells surrounding a 
suppurative center; skin lesions show pseudoepiiheliomatous 
hyperplasia very similar to squamous cell carcinoma. 



Treatment Itraconazole is treatment of choice in most patients; 

amphotericin B, fluconazole, and kctocona/.ole are alternative 
drugs. 



Discussion 



Blastomycosis is a systemic mycotic infection ol humans and 
dogs that is characterized b\ suppuration and granulomatous 
lesions and is caused by the dimorphic fungus Blastomyces 
dermatitidis, the disease is endemic in the southeastern and 
south-central pordons of the United States, and several po< ki is 
of infection extend north along the Mississippi and Ohio rivers 
into central Canada. Clinical disease most commonly involves 
the lungs (acquired by spore inhalation) and then, by 
hematogenous dissemination, the skin, the skeletal system, and 
the male genitourinary tract. Infection cannot be passed from 
person to persi in. 



Atlas Link LIT LJ M-Ml-065 



BLASTOMYCOSIS 



o 

— t 

I — I 

o 

cz 
on 

i — i 
oo 
m 

> 
go 



ID/CC A 25-year-old male presents with sudden-onset double vision 
(DIPLOPIA), dry mouth, weakness, dysarthria, and dysphagia. 

HPI He has no previous history of episodic weakness or of dog or 
rick bites (vs. myasthenia gravis, rabies, or Lyme disease). Last 
night, he ate some home-canned food. 

PE VS: no fever. PE: patient alert; ptosis; bilateral third and tenth 
cranial nerve palsy; symmetric flaccid paralysis of all four limbs; 
deep tendon reflexes reduced; no sensory loss seen; decreased 
bowel sounds. 

Labs Botulinum toxin detected in patient's serum and canned-food 
sample with specific antiserum. 

Treatment Antitoxin; close monitoring of respiratory status; intubation for 
respiratory failure. 



Discussion 



The disease is characterized by gradual return of muscle 
sircngth in most cases. Botulinum toxin is a zinc 
metalloprotease that cleaves specific components of synaptic 
vesicle docking and fusion complexes, thus inhibiting the 
release of acetylcholine at the neuromuscular junction. The 
disease in adults is due to ingestion of the toxin rather than to 
bacterial infection. Botulism is also seen in infants secondary to 
the ingestion of Clostridium bolulinuin spores in honey. 



BOTULISM 



ID/CC A 28-year-old while male visits his family doctor complaining of 
acute pain in both hip joints together with weakness, backache, 
myalgias, arthralgias, and undulating fever of 2 months' 
duration; this morning he woke up with pain in his right testicle. 

HPI For the past 3 years he has worked ai the largest dairy farm in 
his slate. He enjoys drinking "crude" milk. 

PE VS: fever. PE: pallor; marked pain on palpation of sacroiliac 
joints; mild splenomegaly; generalized lymphadenopathy. 

Labs CBC: relative lymphocytosis with normal WBC count. Positive 
agglutination titer (> 1:160); rising serologic titer over time; 
small gram-negative rod Brucella abortus on blood culture. 

Imaging XR, hips: joint effusion and soft tissue swelling without 
destruction. MR, spine: evidence of spondylitis. 

Gross Pathology Lymphadenopathy and splenomegaly; hepatomegaly rare. 

Micro Pathology Granulomatous foci in spleen, liver, and lymph nodes, with 

proliferation of macrophages; epithelioid and giant cells may be 



Treatment Combination therapy with doxycycline or TMP-SMX and 
rifampin or streptomycin. 

Discussion Also called Malta fever, a microbial disease of animals, brucellosis 
is caused by several species of Brucella, a gram-negative, aerobic 
coccobacillus. It is transmitted to humans through the drinking 
ol Contaminated milk or through direct contact with products 
or i issues from animals such as goats, sheep, camels, cows, hogs, 
and dogs. The clinical picture is often vague; thus, a high index 
of suspicion may be necessary for diagnosis. 



r-> 

—i 
i — i 
o 
cz 
co 

1—1 

m 



BRUCELLOSIS 



ID/CC A 26-year-old female presents to the ER with intense, acute-onset 
left lower quadrant crampy abdominal pain, foul-smelling stools 
with streaks of blood, urgency, tenesmus, and fever. 

HPI For the past 2 days, the patient has also had headaches and 

myalgias. She frequently drinks unpasteurized ("raw") milk thai 
she buys at a health-food store. 

PE VS: fever (39°C); tachycardia; normal RR and BR PE: no 

dehydration; diffuse abdominal tenderness more marked in left 
lower quadrant. 

Labs Stool smear shows leukocytes (due to invasive tissue damage in 
I he colon) and gram-negative, curved bacilli, often in pairs, in 
"gull-wing"-shaped pattern; dark-field exam shows motility; 
culture in microaerophilic, 42°C conditions on special agar 
yields Campylobacter jejuni, indicated by oxidase and catalase 
positivity. 

Gross Pathology Friable colonic mucosa. 



Micro Pathology 



Treatment 



Discussion 



Nonspecific inflammatory reaction consisting of neutrophils, 
lymphocytes and plasma cells with hyperemia, edema and 
damage to epithelium, glandular degeneration, ulcerations, and 
crypt abscesses caused by colonic tissue invasion of the 
organism. 

Self-limiting disease. Severe cases (i.e., high fever, severe 
diarrhea) can be treated with fluoroquinolones. 

One of the primary causes of "traveler's diarrhea." Sources of 
infection include undercooked food and contact with infected 
animals and their excreta. Prevent by improving public 
sanitation, pasteurizing milk, and proper cooking. 



CAMPYLOBACTER ENTERITIS 



ID/CC A 49-year-old morbidly obese, diabetic woman presents with 
pruritus in the skin folds beneath her breasts. 

HPI She admits to having this problem chronically, especially in the 
warm summer months, when she perspires more heavily. 

PE Superficially denuded, beefy-red areas beneath breasts with 
satellite vesicopustules and whitish curd-like concretions on 
surface. 

Labs ( 'lusters of budding cells with short hyphae seen under high- 
power lens after skin scales have been put in 10% KOH; 
Candida albicans isolated in Sabouraud's medium. 



o 
c 
l/l 

o 

I— I 

m 



Gross Pathology 



Rash has whitish-creamy pseudomembrane that covers an 
erythematous surface. 



Micro Pathology Yeast invades superficial layers of epithelium. 

Treatment Keep affected areas dry; clotrimazole or other antifungal agents 
locally. 

Discussion Other superficial areas of infection include the oral mucosa 
(thrush), vaginal mucosa (vaginitis), and esophagus (GI 
candidiasis). Systemic invasive candidiasis may be seen with 
immunosuppression, in patients receiving chronic broad-spectrum 
antibiotics, in AIDS patients, or in those receiving 
hyperalimentation. 



CANDIDIASIS 









ID/CC A 25-year-old female presents with painful lumps in her right 
axilla and neck together with low-grade fever. 

HPI Three weeks ago she was scratched on her right forearm by her 
pet cat; an erythematous pustule initially developed at the site 
but resolved spontaneously within 10 days. 

PE VS: fever. PE: tender right axillary and cervical 
lymphadenopathy. 

Labs Lymph node biopsy diagnostic; serologic indirect immunofluo- 
rescent antibody test for Bartonella henselae is positive. 



Micro Pathology 



Hematoxylin and eosin staining reveals granulomatous 
pathology with stellate necrosis and surrounding palisades of 
histiocytic cells; Warthin-Starry silver stain reveals clumps of 
pleomorphic, strongly argyrophilic bacilli. 



Treatment Symptomatic; fluctuant node may need aspiration; azithromycin 
given to immunocompromised patients. 

Discussion Bartonella henselae is the agent that causes cat-scratch disease. 
Lymphadenopathy can persist for months and can sometimes 
be mistaken for a malignancy. Individuals who are 
immunocompromised may present with seizures, coma, 
and meningitis. 



CAT-SCRATCH DISEASE 



ID/CC An 8-year-old white female enters the emergency room 

complaining of headache, malaise, and bipalpebral swelling of 
the right eye. 

HPI She recently returned from a year-long stay in Brazil, where her 
father works as a logger in the Amazon forest. Over the past 
week she had a high fever, which was treated at home as malaria. 

PE VS: fever (39°C); tachycardia. PE: right eyelid swollen shut 

(Romania's SIGN); markedly hyperemic conjunctiva; ipsilateral 
retroauricular and cervical lymph nodes; hepatosplenomegaly. 

Labs PBS: trypanosomes on thick blood smear. ECG: right bundle- 
branch block; ventricular extrasystoles. 

Gross Pathology Encapsulated, nodular area (chagoma) or Romana's sign may 
be seen at point of entry, commonly the face. 



Micro Pathology 



Intense neutrophilic infiltrate with abundant macrophages at 
site of entry: myocardial necrosis with mononuclear cell 
infiltration; pseudocysts in infected rissues contain parasites thai 
multiply within cells; denervation of myenteric gut plexus. 



Treatment X'ifurtimox for acute disease. 

Discussion C'.hagas' disease is a parasitic disease that is restricted to the 
Americas (endemic in South and Central America) and is 
produced by Trypanosoma cruzi. a thin, undulating flagellated 
protozoan; it is transmitted by contamination of a reduviid 
bug bite with injection of its feces. Also known as American 
trypanosomiasis. Long-standing cases show myocardial 
involvement with dilated cardiomyopathy, life-threatening 
conduction defects, and apical aneurysm formation and may 
also show megaesophagus or megacolon. 

Atlas Link DEUE M-Ml-071 



a 

o 

cz: 
on 

o 

i — i 

00 

rri 
> 



CHAGAS' DISEASE 



ID/CC A 35-year-old male complains of cough productive of 
mucopurulent sputum and breathlessness. 

HPI Before the onset of these symptoms, he had a sore throat with 
hoarseness. He has no history of hemoptysis, sharp chest pain, 
or high-grade fever. 

PE Crepitations heard over left lung base. 

Labs CBC: normal leukocyte count. Sputum exam revealed no bacterial 
organism: microimmunofluorescence detected species-specific 
antibodies directed against Chlamydia outer-membrane proteins; 
cultivation of C. pneumoniae demonstrated on HEp-2 and HL 
cell lines. 

Imaging GXR: left lower lobe subsegmental infiltrate with interstitial 
pattern. 

Treatment Doxycycline is the drug of choice; erythromycin and 
fluoroquinolones may also be used. 

Discussion The peak incidence of chlamydia pneumonia is in young adults. 
The mode of transmission would appear to be from person to 
person. 



CHLAMYDIA PNEUMONIA 



ID/CC An 8-year-old male who recently emigrated from India presents 
with bilateral eye irritation and photophobia. 

HPI He reports recurrent episodes of similar eye irritation and 
redness in the past. 

PE Conjunctival congestion; multiple (> 5) follicles, each at least 
0.5 mm in diameter, seen in upper tarsal conjunctiva; 
inflammatory thickening of tarsal conjunctiva; new vessels 
(pannus) seen in cornea at superior limbns; punctate keratitis. 

Labs Diagnosis confirmed by demonstration of characteristic 

cytoplasmic inclusion bodies (Halberstaedter-Prowa/kk BODIES) 
in Giemsa staining of conjunctival scrapings. 



Micro Pathology 



Chlamydia trachomatis is typically seen in conjunctival scrapings in 
colony form in the epithelial cells as H-P inclusion bodies. 
I lisiologically there is lymphocytic infiltration involving the 
whole adenoid layer of parts of the conjunctiva; special 
aggregations of lymphocytes form follicles that tend to show 
necrosis and certain large multinucleated cells (Leber's cells). 



Treatment Topical tetracycline with systemic tetracycline/doxycycline/ 

erythromycin/azithromycin; prophylaxis of family contacts with 
topical tetracycline'. 

Discussion Chlamydia trachomatis causes a variety of ocular diseases, including 
neonatal inclusion conjunctivitis, sporadic inclusion conjunctivitis 
in adults, and sporadic as well as endemic trachoma: trachoma is 
endemic in North Anica, in the Middle East, and among the 
Native American population of the southwestern United States. 
In endemic areas, trachoma is transmitted from eye to hand to 
eye. especially among young i hildren in regions where standards 
of cleanliness are poor. Sporadic U'achoma infection in 
nonendemic areas as well as sporadic inclusion conjunctivitis in 
adults results from transmission of the agent from the genital 
tract to the eye. Trachoma is a major cause of blindness in 
endemic areas. 

Atlas Link mcra 1 1 m-mi-073 



CHLAMYDIA TRACHOMATIS 



— i 

o 

cr 
l/l 

o 

I — I 

m 
> 
t/1 









ID/CC A 30-year-old man has sudden severe, profuse (several liters per 
day) watery diarrhea, protracted vomiting, and abdominal pain. 

HPI He has just returned from a trip to rural India. 

PE Severe dehydration; low urine output; generalized mild abdominal 
tenderness with no signs of peritoneal irritation; stools have 
characteristic "rice-water" appearance; (gray, slightly cloudy 
fluid with flecks of mucus), with no blood. 

Labs Stool culture reveals gram-negative rods with "darting motility"; 
Ol antigen detected; Vibrio cholerae isolated on culture media; 
serum chloride levels decreased; serum sodium levels increased. 

Treatment Vigorous rehydration therapy with oral and/or IV fluids; 
tetracycline, ciprofloxacin, or doxycycbne. 

Discussion A heat-labile exotoxin produced by Vibrio cholerae thai acts by 
permanently stimulating G s protein via ADP ribosylation. 
resulting in activation of intracellular adenylate cyclase, which in 
turn increases cAMP levels and produces secretory diarrhea. 



CHOLERA 



ID/CC A newborn baby is referred to the pediatrician for further 

evaluation of an unusually small head, low birth weight, and an 
extensive erythematous rash. 

HPI Intrauterine growth retardation was prenatal ly diagnosed on 

ultrasound. The child's mother had a flulike episode during the 
first trimester of her pregnancy. 

PE Small for gestational age; generalized hypotonia with sluggish 
neonatal reflexes; extensive "pinpoint" petechial skin rash 
(MULBERRY MUFFIN RASH); microcephaly; chorioretinitis; mild 
icterus; hepatosplenomegaly; sensorineural hearing loss in right 



Labs CBC/PBS: mild thrombocytopenia; atypical lymphocytosis. 

Moderately elevated direct serum bilirubin and transaminases. 
UA: cells in urine found to have large intranuclear inclusions 
(owl's eye inc.i.i skins); cytomegalovirus isolated on tissue 
culture. 

Imaging XR/CT, head: periventricular calcifications; microcephaly. 

Treatment Ganciclovir (only for immunocompromised patients). 

Discussion A congenital herpesvirus infection involving the CNS with eye 
and ear damage, congenital cytomegalovirus is a common cause 
of mental retardation. 



CMV— CONGENITAL 



o 

—i 

o 

cz 
<•> 

o 

H-l 

m 

> 






ID/CC A 1 3-year-old white female visits her pediatrician complaining of 
fever, severe dyspnea, and a dry cough. 

HPI She was recently diagnosed with acute lymphocytic leukemia, for 
which she received a bone marrow transplant. She is currently 
on immunosuppressive therapy. 

PE VS: fever; tachypnea. PE: pallor; crepitant rales over both lung 
fields; mild cyanosis: no hepatosplenomegaly. 

Labs CBC/PBS: anemia; leukopenia. ABGs: hypoxemia. No organism 
in induced sputum stained with Gram, Giemsa, ZN, and 
methenamine silver. 



Imaging 
Gross Pathology 
Micro Pathology 

Treatment 
Discussion 



CXR: diffuse, bilateral interstitial infiltrates. 
Interstitial pneumonitis; hepatitis. 

Characteristic intranuclear inclusions with surrounding halo 

(OWL'S- OR 1st 1 .1 's-ivi i ki is) on transbronchial lung biopsy. 

Ganciclovir (CMV is resistant to acyclovir). 

An enveloped, double-stranded DNA virus belonging to die 
In' i pesvirus group; die mosi c< mimon cause ol pneumonia and 
death in bone marrow transplant patients. It is also common in 
AIDS patients. 



Atlas Link U_D_. M-Ml-076 



CMV PNEUMONITIS 






ID/CC A 30-year-old homosexual white male presents to his family 
physician with a rapidly progressive diminution of vision. 

HPI He is known to be HIV positive and periodically comes in for 
checkups. 

PE Cotton-wool exudates, necrotizing retinitis, and perivascular 
hemorrhages on funduscopic exam. 

Treatment Ganciclovir; foscarnet (CMV is resistant to acyclovir). 

Discussion CMV retinitis is an important treatable cause of blindness that 
occurs in 20% of AIDS patients; 50% to 60% of patients develop 
retinal detachment within 1 year. Toxoplasmosis and progressive 
multifocal leukoencephalopathy (PML) are other important 
causes of blindness in AIDS patients. 



o 
—i 
i — i 
o 

cz 

LO 

i — i 

cn 

m 

> 



CMV RETINITIS 



ID/CC A 19-year-old migrant worker from the southwestern 

United States is brought to the family doctor complaining of 
cough, pleuritic chest pain, fever, and malaise. 

HPI He also complains of a backache and headache along with an 
erythematous skin rash (due to hypersensitivity reaction) in his 
lower limbs. 

PE VS: fever; tachypnea. PK: central trachea; coarse, crepitant rales 
over both lung bases; tender, erythematous nodules over shins 

(l.RYim-.MA \oiiosr\i); periarticular swelling of knees and ankles. 

Labs Positive skin test with coccidioidin; dimorphic fungi (hyphae in 
soil; spherules in body tissue); Coccidioides immitis on silver stain 
and sputum culture; positive latex agglutination test. CBC/PBS: 
eosinophilia. 

Imaging CXR: nodular infiltrates and thin-walled cavities in both lower 
lungs. 

Gross Pathology Caseating granulomas; often subpleural and in lower lobes; 
necrosis and cavitation may also be present. 

Micro Pathology Silver-stained tissue sections show spherules filled with 
endospores. 

Treatment Amphotericin II or itraconazole. 

Discussion Endemic in the southwestern United States, coccidioidomycosis 
is produced by C. immitis and is transmitted by inhalation of 
arthrospores. Systemic dissemination is frequent in blacks as 
well as in immunosuppressed and pregnant patients. Meningitis 
or granulomatous lung disease may result, which may lead to 
death. 



COCCIDIOIDOMYCOSIS 









ID/CC A 28-year-old male who lives in the northwestern United States 
complains of a high-grade fever with rigors, generalized aches, 
myalgias, headache, and backache. 

HPI Four days ago In- relumed from a hiking trip during which he 
was bitten by a tick: he took amoxicillin as prophylaxis against 
Lyme disease. 

PE VS: fever. 

Labs CBC: leukopenia; relative lymphocytosis. Viral antigen detected 
in RBCs by immunofluorescence: Colorado tick virus cultured 
in suckling mice by intracerebral inoculation of blood clot; 
indirect fluorescent Ab test positive. 

Treatment Symptomatic. 



5 

Tl 

m 
(~i 

—l 

O 

cz 
on 

2 

m 
> 
oo 






Discussion Colorado tick fever virus is an 80-nm double-shelled reovirus 
thai is covered with capsomeres; its icosahedral core contains 
1 2 segments of dsRNA. The disease is a zoonosis that is 
transmitted by a wood tick, Dermacentor andersoni. It occurs 
primarily in the Rocky Mountain region, primarily affecting 
hikers. Since no specific therapy exists, prevention is key (wear 
clothing diat covers the body). 



COLORADO TICK FEVER 



ID/CC A 2-year-old male is brought to the F.R by his parents with sore 
throat, inspiratory stridor, and a barking cough of 1 day's 
duration. 

HPI The patient has no significant past medical history. 

PE VS: fever (38.6°C); tachypnea. PE: respiratory distress; 

nasopharyngeal discharge; diffuse rhonchi and wheezes; 
examination of extremities reveals some cyanosis. 

Labs Throat and nasal swabs isolate parainfluenza virus; serocliagnosis 
and hemagglutinin inhibition tests reveal type 1 (most common 

cause) . 

Imaging CXR: air trapping. XR, neck: subglottic narrowing. 

Gross Pathology Inflammation and edema of larynx, trachea, and bronchi. 

Treatment Most cases respond to supportive therapy such as humidified air, 
removal of secretions, and bed rest. Severe cases may require 
humidified oxygen, racemic epinephrine, or high-dose 
corticosteroids. 

Discussion Differentiate croup from Htiriiwp/i/hi.s influenzae type B and 
influenza A virus. Modes of transmission include respiratory 
droplets and person-to-person contact; tends to peak in the fall 
and winter. Most cases of croup are due to parainfluenza virus 
ivpe 1 ; type 3 is a prominent cause of bronchiolitis in babies. 



CROUP 



ID/CC A 30-year-old man with AIDS presents with chronic, recurrent 
profuse, nonbloody, watery diarrhea. 

HPI The diarrhea has recurred over the past 2 months with 
intermittent cramping, and previous treatments have not 
been effective. 



PE VS: no fever. PS: moderate dehydration; thin; generalized 
lymphadenopalhy. 

Labs Acid-fast staining demonstrates oocysts of Cryptosporidium in 
fresh stool. 



Gross Pathology 
Micro Pathology 



Intestinal mucosa appears normal. 

Blunting of intestinal villi; mixed inflammatory cell infiltrates 
with eosinophils in lamina propria; organisms visible on brush 
borders. 



— j 
i— i 
o 

cr 
u-> 

o 
i—i 

C/1 



te 



Treatment No treatment found effective; supportive management with 
maintenance of fluids and nutrition. 



Discussion Cryptosporidium parvum infection presents as acute diarrhea 
in malnourished children and as severe diarrhea in 
immunocompromised patients (part of HIV wasting syndrome); 
tin- disease is mild and self-limiting in immune-competent 
patients. The disease is acquired through the ingestion of 
oocysts (fecal-oral transmission) that may be killed by 
chlori nation. 



Atlas Link 



:OZ1 M-Ml-081 



CRYPTOSPORIDIOSIS 






ID/CC A 5-year-old white male presents with malaise, anorexia, low- 
grade fever, sore throat of 3 days' duration, and dyspnea on 
exertion. 

HPI The child was raised abroad. His immunization status cannot be 
determined. 

PE VS: fever; tachycardia with occasional dropped beats. PE: 

cervical lymphadenopathy (bui.i.'s-nkck. appearance); smooth, 
whitish-gray, adherent membrane over tonsils and pharynx; no 

hepatosplenomegaly; diminished intensity of SI. 

Labs Metachromatic granules in bacilli arranged in "Chinese character" 
pattern on Albert stain of throat culture; C.orynebacterium 
diphtheriae confirmed by growth observed on Ldffler's blood 
agar; erythema and necrosis following intradermal injection of 
(',. diphtheriae toxin (POSITIVE Schick's test); immunodiffusion 
studies (Elek's) confirm toxigenic strains of C. diphtheriae. ECG: 
ST-segment elevation; second-degree heart block. 

Imaging Echo: evidence of myocarditis. 



Gross Pathology 



Micro Pathology 



Treatment 



Discussion 



Pharyngeal membranes not restricted to anatomic landmarks; 
pale and enlarged heart. 

Polymorphonuclear exudate with bacteria: precipitated fibrin 
and cell debris forming a pseudomembrane; marked hyperemia, 
edema, and necrosis of upper respiratory tract mucosa; 
exotoxin-induced myofibrillar hyaline degeneration; lysis of 
myelin sheath. 

Begin treatment on presumptive diagnosis; specific antitoxin 
and penicillin or erythromycin; respiratory and cardiac support; 
confirm eradication by repeating throat culture. 

A bacterial infection of the throat, diphtheria is preventable by 
vaccine and is caused by toxigenic Corynebacteriwn diphtheriae, a 
club-shaped, gram-positive aerobic bacillus. Diphtheria toxin is 
produced by B-pi ophage-infected corynebacteria; it blocks EF-2 
via ADP ribosylation and hence ribosomal function in protein 
synthesis. The toxin enters the bloodstream, causing fever, 
myocarditis (within the first 2 weeks), and polyneuritis (many 
weeks later) . 



Atlas Links UJCta » I M-Ml-082 



DIPHTHERIA 



MTl MC-324 









ID/CC A 5(>-ycar-old male professor of veterinary medicine from 
New Zealand experiences sudden high fever with chills, 
jaundice, and right upper quadrant pain while attending a 
conference in the United Stales. 



HPI I lis past history is unremarkable. He has been healthy and has 
been physically active working in the field with sheep and 
breeding dogs. 

PE VS: fever; hypotension (BP 90/50). PE: hepatomegaly; jaundiced 
sclera; on palpation of epigastrium and right hypochondrium, 
abdomen is tender with no rebound tenderness. 

Labs CBC: leukocytosis with neutrophilia; slight eosinophilia. 

Strongly positive iinmunoblot test for antibodies to echinococcal 
antigens; elevated direct bilirubin and alkaline phosphatase. 



Imaging 



Gross Pathology 



CT/US, abdomen: multiple large septated liver cysts impinging 
on bile ducts, producing biliary dilatation (due to obstruction). 

Liver is most common site of invasion, but cysts may also form in 
lungs, kidney, bone, and brain; each cyst contains millions of 
scoleces and consists of two layers: an inner germinal layer and 
an outer laminated layer; usually surrounded by fibrotic 
reaction. 



o 

—I 
i — i 
o 
cz 
oo 

o 

I— I 

on 
m 
> 



Micro Pathology Giant cell reaction surrounding cyst with eosinophilic 
infiltration. 

Treatment Surgically remove cysts if possible; albendazole may be effective. 

Discussion Echinococcosis is a zoonosis produced by Echinococcus 

granulosus. It is acquired through the ingestion of food or drink 
contaminated with the feces of dogs or odier carnivores that 
have eaten contaminated meat; humans are die intermediate 
host of parasitic larvae. Accidental spilling of cyst fluid, either 
spontaneously or during surgery, may result in secondary 
seeding or anaphylaxis and even death. Also known as hydatid 
disease. 






Atlas Link ID. 



M-Ml-083 



ECHINOCOCCOSIS 



ID/CC A 28-year-old male who is a resident of the southeastern 

United States presents with a high fever with chills, headache, 
and myalgias. 

HPI He remembers having been bitten by a tick a week before 
developing his symptoms; however, he reports no skin rash. 

PE VS: fever. PE: no skin rash noted. 

Labs CBC: leukopenia and mild thrombocytopenia. Characteristic 
intraleukocytic inclusion bodies and serologic response to 
Ehrlichia antigens demonstrated; E. chaffeensis cultured from 
blood and detected by PCR. 

Treatment Doxycycline. 

Discussion Ehrlichieae are gram-negative, obligately intracellular bacteria. 
The two types of Ehrlichia species that affect humans are 
/■,'. chaffeensis (which attacks macrophages and monocytes) and 
an /.'. equWke organism (which attacks granulocytes). Preventive 
measures include wearing clothing that covers the body and 
using insect repellants. 



EHRLICHIOSIS 






ID/CC A 30-year-old male from Texas presents with fever and a skin 
rash thai began about 2 weeks ago. 

HPI The onset was gradual, with prodromal symptoms of headache, 
malaise, backache, and chills. These symptoms were followed by 
shaking chills, fever, and a more severe headache accompanied 
by nausea and vomiting. A remittent pattern of fever 
accompanied by tachycardia continued for 10 to 12 days, with 
the rash appearing around the fifth day of fever. The patient 
worked at a rat-infested food-storage depot this summer. 

PE VS: fever. PE: discrete, irregular pink maculopapular rash seen 
in axillae and on trunk, thighs, and upper arms; face, palms, 
and soles only sparselv involved: mild splenomegaly noted. 

Labs The Weil-Felix agglutination reaction for Proteus strain OX-19 
was positive; complement-fixing antibodies to the typhus group 
antigen were demonstrated; endemic typhus (due to Rickettsia 
typhi) was confirmed serologically by using specific washed 
rickettsial antigens in IFA tests. 






~n 
m 

Q 

i — i 

o 
cz 
oo 

o 

I — I 

on 

m 
3> 
oo 
m 



Treatment Antibiotic treatment with doxycycline (chloramphenicol is used 
as an alternative). 

Discussion Murine typhus is a natural infection of rats and mice by 

Rickettsia typhi; spread of infection to humans by the rat flea is 
incidental and occurs when feces from infected fleas are 
scratched into the lesion. Cases can occur year-round; however, 
most occur during the summer months, primarily in southern 
Texas and California. 



ENDEMIC TYPHUS 









ID/CC A 28-year-old Guatemalan male is brought to the hospital 

complaining of severe headache, photophobia, and fever over 
the past 2 weeks. 

HPI As a political dissident, he spent 4 months in a refugee camp in 
soi u hern Mexico before entering the United States. 

PE VS: fever (40°C). PE: papilledema and delirium; bilateral 
swelling of parotid glands 1 week later; toxic facies; 
maeulopapular rash on trunk and extremities; face, palms, 
and soles spared; mild splenomegaly- 
Labs Positive Weil-Felix reaction to OX-1 9 strains of Proteus; rise in 
complement fixation titer for Rickettsia prowazekii; specific 
antibodies. UA: proteinuria; microscopic hematuria. 

Gross Pathology Myocarditis and pneumonia may be present; cerebral edema; 
maeulopapular rash. 



Micro Pathology 



Zenker's degeneration of striated muscle: thrombosis and 
endothelial proliferation of capillaries with abundant rickettsiae 
and perivascular cuffing; accumulation of lymphocytes; 
microglia and macrophages (typhus nodules) in brain. 



Treatment Doxycycline; chloramphenicol. 

Discussion Epidemic typhus is a febrile illness caused by Rickettsia prowazekii, 
a gram-negative, nonmotile, obligate intracellular parasite; it is 
transmitted via body lice and is associated with war, famine, and 
crowded living conditions. The rash should be differentiated 
from Rocky Mountain spotted fever, which starts peripherally on 
the wrists and ankles and also includes the palms and soles. 

Atlas Link L'jciazi MC-169 



EPIDEMIC TYPHUS 



o 



ID/CC A 4-year-old male presents with fever, hoarseness, and 

respiratory distress because of partial airway obstruction. 

HPI Tlie child is also unable to speak clearly and has pain while 
swallowing (oiiym iphagia) . 

PE VS: fever; tachypnea. PE: patient is leaning forward wiUi neck 
hyperextended and chin protruding; drooling; marked 
suprasternal and infrasternal retraction of chest: inspiratory ^ 

stridor on auscultation. cz 

CO 

Labs Culture of throat swab (no role in management of acute to 

disease) reveals penicillinase-resislant Haemophilus influenzae; > 

blood cultures also positive. m 

Imaging XR, neck: marked edema of epiglottis and aryepiglottic folds 
("thumbs-up" sign). 

Gross Pathology Epiglottis is cherry-red, swollen, and "angry-looking." Rapid 

cellulitis of epiglottis and surrounding tissue leads to progressive 
blockage of airway. 

Treatment Preservation of airway; IV cefuroxime. 

Discussion The principal cause of acute epiglottitis in children and adults is 
H. influenzae type b; other pathogens include II. parainfluenza 
and group A streptococcus. Characterized by rapid onset. 

Atlas Link "TFI" PG-M1-087 



EPIGLOTTITIS 



ID/CC A 30-year-old soldier who had been admitted for a gunshot 

wound in the right ihigli presents with severe pain and swelling 
at the site of his injury. 

HPI The patients right lower limb had become discolored, and 
several bullae had appeared on the skin. He has passed very 
Utile urine over the past day. and the urine he has passed has 
been dark ("cola-colored"). 

PE VS: low-grade fever; marked tachycardia. PE: diaphoresis; skin 
of right thigh discolored (bronze to purple red); site of injury 
exquisitely tender and tense and oozing a thin, dark, and 
foul-smelling fluid; crepitus while palpating thigh. 

Labs CBC: low hematocrit. Gram stain of exudate and necrotic 

material at wound site reveals presence of large gram-positive 
rods; anaerobic culture of exudate and blood yields Clostridium 
perfringens type A: culture isolate demonstrates positive Nagler 
reaction (due to presence of alpha toxin lecithinase): further 
labs confirm presence of intravascular hemolysis, myo- and 
hemoglobinuria, and acute tubular necrosis. 

Imaging XR. right thigh: presence of gas in soft tissues. 

Gross Pathology Overlying skin purple-bronze, markedly edematous with 
vesiculobullous changes with little suppurative reaction. 



Micro Pathology 



Treatment 



Discussion 



Coagulative necrosis, edema, gas formation, and many large 
gram-positive bacilli found in affected muscle tissue; relatively- 
sparse infiltration of PMNs noted in the bordering muscle 
tissue. 

Surgical debridement: antibiotics (penicillin, clindamycin. 
tetracycline, metronidazole); hyperbaric oxygen therapy and 
polyvalent antitoxin; supportive management of associated 
multiorgan failure. 

A rapidly progressive myonecrosis caused by Clostridium 
/irr/ringens type A, traumatic gas gangrene develops in a wound 
with low oxygen tension (embedded foreign bodies containing 
calcium or silicates cause lowering of oxygen tension, leading to 
germination of the spores). The most important toxin is the alpha 
toxin leciihinase, which produces hemolysis and myonecrosis. 



Atlas Link MIL, PG-M1-088 



GAS GANGRENE— TRAUMATIC 



ID/CC A 4-year-old female is brought to the pediatrician because of 
lack of appetite; nausea and vomiting; chronic, foul-smelling 
diarrhea without blood or mucus; and a bloated sensation. 

HPI She has been in several day-care centers over the past 3 years. 

PE Low weight and height for age; mild epigastric tenderness. 

Labs Binucleate, pear-shaped, flagellated trophozoites {GiARDlA 

lAMiu.i.x) on freshly passed stool; cysts found on stool exam. 

Treatment Metronidazole. 

Discussion The most common protozoal infection in children in the United 
States, giardiasis is transmitted mainly through contaminated 
food or water and causes malabsorption. 

Atlas Links ETIcra \ I M-M1-089A, M-M1-089B 



o 

— i 

o 

c: 
co 

o 
I— I 

CO 

m 
> 



GIARDIASIS 



ID/CC A 3-day-old female neonate presents with a thick eye discharge. 

HPI The mother admits to having multiple sexual partners and 

complains of a vaginal discharge. She did not receive adequate 
antenatal care. 

PE Exam of both eyes reveals a thick purulent discharge and 
marked conjunctival congestion and edema; conjunctival 
chemosis is so marked that cornea is seen at bottom of a 
c rater-like pit; corneal ulceration noted. 

Labs Conjunctival swabs on Gram staining reveal presence of gram- 
negative diplococci both intra- and extracellularly in addition to 
many PMNs; conjunctival swab and maternal cervical culture 
yield Neisseria gonorrhoeae. 

Treatment Aqueous penicillin G or ceftriaxone for a total of 7 days. Also 
treat mother and her sexual contacts. Educate the mother 
regarding the importance of safe sex. 

Discussion Caused by Neisseria gonorrhoeae, gonococcal ophthalmia 

neonatorum is contracted from a mother with gonorrhea as the 
fetus passes down the birth canal; infection does not occur in 
utero. Corneal inflammation is the major clinical sign that may 
produce complications such as corneal opacities, perforation, 
anterior synechiae, anterior staphyloma, and panophthalmitis. It 
is now common practice to prevent this disease by treating the 
eyes of the newborn with an antibacterial compound such as 
erythromycin ointment or 1% silver nitrate; however, home 
childbirth bypasses this prophylactic procedure, and thus some 
cases are still occurring in the United States. 



GONOCOCCAL OPHTHALMIA NEONATORUM 






ID/CC A 19-year-old white male presents with burning urination; 

profuse, greenish-yellow, purulent urethral discharge; staining 
of his underwear; and urethral pain. 

HPI Four days ago, he had unprotected sexual contact with a 
prostitute. 

PE Mucopurulent and slightly blood-tinged urethral discharge; 
normal testes and epididymis; no urinary retention. 

Labs Smear of urethral discharge reveals intracellular gram-negative 
diplococci in WR( is; gonococcal infection confirmed by 
inoculation into Thayer-Martin medium. 

Gross Pathology Abundant, purulent urethral exudate. 

Treatment Ceftriaxone plus doxycycline or erythromycin for possible 
coinfection with Chlamydia. 

Discussion A common STD caused by Neisseria gonorrhoeae, gonorrhea may 
involve the throat, anus, rectum, epididymis, cervix, fallopian 
tubes, prostate, and joints; conjunctivitis is also found in 
neonates. Neonatal conjunctivitis may be prevented through the 
instillation of silver nitrate or erythromycin eye drops at birth. 



o 

—i 
i — i 
o 
c= 
on 

O 
i — i 
GO 
m 
3> 
ir, 



Atlas Links H'lcyjn M-Ml-091 



IM2-018 



GONORRHEA 






ID/CC A 28-year-old male immigrant presents with inguinal swelling 
and a painless penile ulcer. 

HPI He admits to unprotected intercourse with multiple sexual 
partners, many of whom were prostitutes. He first noticed a 
papule on his penis several weeks ago. 

PE Soft, painless, raised, raw, beef-colored, smooth granulating 
ulcer noted on distal penis: multiple subcutaneous swellings 

(rsi i now ii( its) noted in inguinal region, some of which have 
ulcerated. 

Labs Giemsa-stained smear from penile and inguinal regions 

demonsU'ate characteristic "closed safety pin" appearance 
i >l en. apsulated organisms within a large histiocyte 
(Donovan bodies). 

Micro Pathology Characteristic histologic picture of donovanosis comprises some 
degree of epithelial hyperplasia at margins of lesions; dense 
plasma cell infiltrate scatters histiocyte-containing Donovan 

b< idies. 

Treatment Treat with doxycycline or double-strength TMP-SMX. 

Discussion Granuloma inguinale, a slowly progressive, ulcerative, 

granulomatous STD involving the genitalia, is caused by the 
gram-negative bacillus Calymmatobacterium granulomatis 
(formerly Donovdnia gmti.ulom/ilis): ii is seen in Giemsa-stained 
sections as a dark-staining, encapsulated, intracellular rod- 
shaped inclusion in macrophages, the so-called Donovan body. 
The disease is endemic in tropical areas such as New Guinea, 
southern India, and southern Africa. 



GRANULOMA INGUINALE 






ID/CC A 60-year-old male presents with cough productive of 

mucopurulent sputum together with mild lever and worsening 
breathlessness. 

HPI He is a chronic smoker who has been diagnosed with COPD. 

PE VS: lever. PE: in moderate respiratory distress; emphysematous 
i hesi with obliterated cardiac and livei dullness; wheezing and 
crackles heard over both lung fields. 

Labs Haemophilus influenzae organisms seen as small, pleomorphic 
gram-negative ba< illi on Gram stain of sputum; nontypahle 
H. influenzae isolated on sputum culture (to grow in culture, 
II influenzae requires both factor X-hematin and factor 
V— nicotinamide nucleoside present in erythrocytes). 

Treatment Amoxicillin/ampicillin therapy; TMP-SMX. azithromycin, and 

clarithromycin arc also excellent drugs for the treatment of 
clinically mild to moderate //. influenzae infections of the upper 
respiratory tract. 

Discussion Infections caused by nontypahle. or unencapsulated, 

Haemophilus influenzae strains have been increasingly recognized 
in pediatric and adult populations. Nontypable H. influenzae 
strains are frequent respiratory tract colonizers in patients with 
COPD and commonly exacerbate chronic bronchitis in these 
patients; nontypable strains are also the most common cause of 
acute otitis media in children. 



H. INFLUENZAE IN A COPD PATIENT 



r~i 



CO 

o 

l—i 

co 

£ 









ID/CC A 25-year-old male presented with sudden-onset breathlessness, 
cough, cyanosis, and high-grade fever. 

HPI The patient failed to improve on 100% oxygen, became 

hypotensive, and died of type 2 respiratory failure a few hours 
after admission. He had been in perfect health and had been 
hiking in several rodent-infested areas before falling ill. 

PE On admission he was found to have fever, tachycardia, cyanosis, 
hypotension, and rales on auscultation over both lung fields; no 
meningeal signs or localizing CNS signs could be demonstrated. 

Labs ABGs: respiratory acidosis with hypoxia and hypercapnia. CBC: 
leukocytosis; hemoconcentration; thrombocytopenia; atypical 
lymphocytosis. Increased LDH and ALT levels; prolonged PT 
index; sputum exam and blood culture did not yield any 
organism; IgM antibody to hantavirus and immunohistochemical 
stains for hantavirus antigen in tissues confirmed infection with 
the virus. 

Imaging CXR: noncardiogenic pulmonary edema (bat-wing edema 
pattern). 



Micro Pathology 



Histopathologic exam of lung tissues was suggestive of acute 
respiratory distress syndrome (adult hyaline membrane disease). 



Treatment Patient died despite intensive ventilatory support (Sin Nombre 
virus most frequently causes hantavirus pulmonary syndrome in 
the United States). 

Discussion A virus closely related to the Hantaan virus (which produces 
Korean hemorrhagic fever and hemorrhagic fever with renal 
syndrome) has been recovered from mice in various regions of 
the United States; rodents are the natural host for this group of 
viruses. Infected rodents shed the virus in saliva, urine, and 
feces for many weeks, and humans are believed to acquire the 
infection via exposure to rodent excrement or saliva, either by 
the aerosol route or by direct inoculation. 



HANTAVIRUS PULMONARY SYNDROME 






ID/CC A 35-year-old male who works as a U.N. health worker presents 
with a high-grade fever and massive hematemesis. 

HPI He recently returned from Zaire, where he worked in a 
lick-infested forest. 

PE VS: fever. PE: extensive ecchymosis. 

Labs CBC: leukopenia; severe thrombocytopenia. LFTs: elevated AST. 
Crimean-Congo virus isolated. 

Treatment Treatment involves a 10-day course of ribavirin; platelet 

transfusions; avoid salicylates; barrier nursing and containment 
of infected secretions, since airborne infection may occur in 
hospital environment. 

Discussion The agent responsible for Crimean-Congo hemorrhagic fever is 
a bunyavirus; reservoirs include wild and domesticated sheep, 
cattle, goats, and hares. The disease is transmitted by a tick 
vector, usually an ixodid of the genus Hyalomma; endemic areas 
include the Middle East and western China. The disease targets 
individuals of all ages and affects males and females equally. 



o 

— i 

o 

c: 
on 

o 
i — i 
tn 
m 
> 

IS) 



HEMORRHAGIC FEVER— CRIMEAN-CONGO 



ID/CC A 10-year-old male is brought to the ER in a slate ol' shock 
accompanied by massive hematemesis. 

HPI The family had just returned from a vacation in Thailand. His 
parents say that he had a high-grade fever for 5 to fi days, for 
which he was receiving presumptive treatment for malaria. 

PE VS: hypotension: tachycardia. V'S: tool, clammy extremities; 
petechial skin rash over extremities, axillae, trunk, and face; 

bleeding from venipuncture sites. 

Labs CBC: thrombocytopenia; hematocrit increased by > 20%. 

Abnormal clotting profile suggestive o I disseminated 
intravascular coagulation (DIC); paired sera reveal significant 
rise in liter of hemagglutination inhibition antibodies against 
Dengue virus serotypes 1 and 2. 

Imaging LIS: bilateral pleural effusion and ascites. 

Treatment Symptomatic; manage shock with fluids and hemodynamic 

monitoring: fresh blood/ plaieiei-rich plasma: avoid salicylates. 

Discussion Dengue hemorrhagic fever is caused by a mosquito-borne {Aedes 
aegypti) flavivirus and is characterized by four distinct dengue 
serotypes (type 2 is considered the most dangerous). A. aegypti 
has a domestic habitat (stagnant water in llovver pots, old jars, tin 
cans, and old tires) and bites during the day. Dengue fever has 
shown an increase in incidence in Southeast Asia, Central and 
South America, and the Caribbean. Since no specific therapy 
exists, prevent by avoiding contact with infected A. aegjpli. 

Atlas Link i L : Z-Ml-096 



HEMORRHAGIC FEVER— DENGUE 






ID/CC A 58-year-old man who was hitchhiking through central and 

southern Africa was admitted to a hospital in /.aire in a slate of 
shock following massive hemorrhage from the GI tract 
(hcmatemesis and melena); he died within 6 hours of 
admission. Ten days later, a male doctor who had attended ihis 
patient and had attempted resuscitation became ill with a 
similar disease syndrome. 

HPI Ai admission, he gave an 8-day hisiorv of progressive fever, 
severe headaches, myalgias, and watery diarrhea. He also 

reported an erythematous, measles-like skin rash that had 
begun to desquamate. 

PE VS: lever. PE: splenomegaly; hepatomegaly. 

Labs CBC: leukopenia; Pelger-1 luet anomaly of neutrophils with 

atypical mononuclear cells: thrombocytopenia widi abnormal 
platelet aggregation. Markedly elevated AST and ALT: blood was 
inoculated intraperitoneal!)' into young guinea pigs and into 
various tissue culture cell lines, and Ebola virus was detected by 
indirect immimofluorescent staining techniques. 

Gross Pathology At autopsy, lymph nodes, liver, and spleen (bund to be most 

conspicuously involved (replication of Ebola virus can occur in 
virtually all organs); stomach and intestines filled with blood; 
petechiae seen over bowel mucosa. 



Micro Pathology 



Treatment 



Discussion 



Severe congestion and stasis of spleen: widespread necrosis of 
liver cells; electron microscopy of liver i cvealed pleomorphic 
virus particles appearing in contrast preparations as long, 
filamentous forms, U-shaped forms, and some circular forms 
resembling a doughnut. 

Supportive care, since no specific treatment exists: a prior 
outbreak was brought under control bv isolating patients and 
instituting Strict barrier nursing. 

A hemorrhagic, febrile infection of humans due to infection with 
the Ebola and Marburg viruses, both of which are liloviruses that 
are structuralh indistinguishable but anligcnicallv distinct. This 
disease is a zoonosis but the reservoir is unknown. Individuals can 
become infected through person-to-person or nosocomial contact 



a 

o 

cr 

GO 

O 
i — i 

go 
m 
> 

CO 






HEMORRHAGIC FEVER— EBOLA VIRUS 



ID/CC A 25-year-old male woodcutter who lives in South Korea is 

admitted to the ER in a state of shock and massive epistaxis. 

HPI The patient had been complaining of fever, malaise, headache, 
myalgias, back pain, abdominal pain, nausea, and vomiting for 
the past week; he also complained of extremely reduced urine 
output. Careful history revealed that before he fell ill. he and his 
friend were cutting wood in the forest when they accidentally 
disturbed a rodent-infested area. 

PE VS: hypotension. PE: epistaxis; facial Hushing; pctechiae and 
subconjunctival hemorrhages. 

Labs Deranged RFTs suggestive of acute renal failure. CBC: 

thrombocytopenia. Serology and culture identify hantavirus, 
Hantaan serotype. 

Treatment Supportive management in the form of dialysis (for renal 

failure); management of shock and hemorrhage; IV ribavirin 

(must start within first 4 days of manifestation of disease). 

Discussion Korean hemorrhagic fever with renal syndrome is caused by the 
Hantaan serotype of hantavirus. Its reservoirs are various 
rodents that are found distributed over Europe and Asia; 
humans acquire the disease mainly by inhaling aerosols of 
rodeni virus. 



HEMORRHAGIC FEVER— RENAL SYNDROME 






ID/CC A 7-year-old male complains of a high fever and a very sore 
throat. 

HPI The pain is so severe thai the child refuses to swallow. He is 
adequately immunized and achieved normal developmental 
milestones. 

PE VS: fever. PE: characteristic grayish-white vesicular lesions, some 
of which have ulcerated, noted over soft palate and tonsils. 

Labs Coxsackievirus A isolated from mucosal lesions. 

Treatment Self-limiting condition. 

Discussion In hand, foot, and mouth disease (HFMD), patients complain 
of fever, weakness, and decreased appetite along with similar 
1 i-M oils noted in tin- oral cavity, palms, soles, and buttocks. 
Herpangina may be caused by coxsackievirus Al— A10. A16, 
A22, and B1-B5. Outbreaks of HFMD are usually caused by 
coxsackievirus A16. 






1/1 

o 

►— i 

m 

> 

on 






HERPANGINA 



ID/CC A 25-year-old homosexual male visits a health clinic complaining 
of headache, low-grade fever, and a painful skin rash in the 
perianal area. 

HPI He has no history of penile ulcerations and admits to 
unprotected anal sex with multiple partners. 

PE Perianal vesicular rash in clusters on erythematous base; no 

penile ulceration; painful inguinal lymphadenopalhy. 



Labs 



Gross Pathology 



Multinucleated giant cells with intranuclear inclusions 

sin rounded by clear halo on Pap-stained section or Tzanck 
preparation of scrapings from base of vesicles. 

Clear liquid in vesicles: secondary bacterial infection may result; 
painful ulcerations when vesicles rupture. 



Micro Pathology Inflammatory infiltrate with abundant lymphocytes. 

Treatment Acyclovir. 

Discussion An enveloped, double-stranded DNA virus transmitted by sexual 
contact, HSV 2 has a tendency to recur and can be transmitted 
to the fetus through the birth canal. Condom use appears to be 
one of the most effective means of preventing transmission. 



Atlas Links mem 1 1 M-Ml-100 



T Li; IM2-019A, IM2-0WB 



HERPES GENITALIS 






ID/CC A 45-year-old HIV-positive male is seen by his family doctor 

following die appearance of a painful, burning skin rash on the 
left side of his chest that is accompanied by a headache and 
low-grade fever. 

HPI The patient had chickenpox as a child. He had been well 

until 1 year ago, when he was diagnosed with non-Hodgkin's 
lymphoma, for which he is etirrendy undergoing chemotherapy. 

PE Vesicular rash on erythematous base; in dermatomal distribution 

(left T6-T8); exquisitely tender to touch. 

Labs Acantholytic cells on Tzanck smear from base of vesicles. 

Micro Pathology Intranuclear eosinophilic inclusions surrounded by clear halo 

(CowdryA INCLUSIONS). 

Treatment Acyclovir. 

Discussion Shingles represents a reactivation of a latent infection with 

varicella-zoster virus; the rash typically follows the distribution 
of a nerve root. It is commonly seen in immunosuppressed 
patients and is also associated with trauma, ultraviolet radiation, 
hypothermia, and emotional stress. Postherpetic neuralgia is a 
common complication in die elderly. 

Atlas Links mcvj2i IM2-020A, IM2-020B 




o 

— i 
i—i 
o 
cz 
00 

o 

I— I 

00 

m 
> 
CO 






HERPES ZOSTER (SHINGLES)